Вы находитесь на странице: 1из 59

www.Xerous.

com
Surgery Online

1/1
American Board of Surgery In service Training
Examination Questions
1. Advantages of Enteral over Parenteral Feedings
Enteral feedings offer a number of advantages over parenteral feeding.
The two primary advantages are:
1. Reduced cost
2. Decreased risk of sepsis
When compared to TPN, newer studies have demonstrated that enteral
nutrients appear to maintain the integrity of the gastrointestinal tract,
and thus reduce the incidence of bacterial translocation from the gut.
An oral diet preserves gut mucosal mass and maintains digestive
enzyme content. Further, oral feedings stimulate the gut to elaborate
trophic hormones, particularly gastrin, and enteral calories initiate
greater insulin release and can thus promote anabolism. The addition
of fiber and glutamine through enteral feedings has proven to be of
significant benefit in maintaining mucosal integrity and increasing the
absorptive capacity (increased length of villi). Glutamine is a major
metabolic fuel for the enterocytes, and may play an important role in
modulating the cytokine release from intestinal lymphocytes. Recent
studies in the MSU Shock Trauma Research Unit have demonstrated
that carbohydrates and lipids are poorly absorbed in animals and
humans following the stress of operation, hemorrhage or sepsis.
Similar reductions in absorption of protein have been noted by other
authors. Immediately following stress, as little as 50% of the enteral
diet is absorbed. This reduced absorptive capacity may last 7-10 days.
Thus, parenteral nutrition may be necessary as a supplement to enteral
feedings to met the nutritional requirements of stressed surgical
patients.

Schwartz, 6th Edition, p. 255

2. Characteristics of Tension Pneumothorax Under General Anesthesia


A tension pneumothorax can develop under general anesthesia.
Etiologic factors include:
A pneumothorax may follow a traumatic intubation in which a
perforation of the pharynx or piriform sinus may have occurred. The
pneumothorax occurring during mechanical ventilation can arise from
a subclavian catheterization in which an occult injury to the pulmonary
parenchyma results in a delayed tension pneumothorax, or in
circumstances where prior pleural biopsy may have occurred. Trauma
www.Xerous.com
Surgery Online

2/2
patients with multiple injuries, including rib fractures or chest wall
contusions, may develop delayed tension pneumothorax during
operations required to correct associated injuries. Pulmonary
barotrauma can lead to the development of a pneumothorax during
positive pressure ventilation.
Pneumothorax has been closely associated with peak inspiratory
pressures, which exceed 50 cm H2O, which is continued for 12 hours
or more. The incidence of pneumothorax is 43% with PIP greater than
70 cm H2O. Tension pneumothorax is present in 50% of ventilatory
associated pneumothoraces.
Characteristics of a pneumothorax during anesthesia include
progressively increasing inspiratory pressures, in patients receiving
volume regulated ventilation. Progressive hypoxia and hypercapnia
followed by hypotension and sudden death follows if undetected.
The diagnosis is established by noting deviation of the trachea,
distention of neck veins, and absent or distant breast sounds on the
affected hemithorax. The hemidiaphragm will be pushed caudally, and
it may be apparent during an upper abdominal operation. Treatment
includes rapid decompression of the tension pneumothorax. Placing a
large bore needle through the chest wall or rapidly placing a chest tube
through the diaphragm or intercostal space should be accomplished
rapidly.

Critical Care: Civetta et al., pp 248, 1183


Schwartz: 6th Ed., pp 177, 705

3. Complications of Cyclosporine Use


Cyclosporine was discovered in 1972, and is a new class of
immunosuppressive agents. It is a cyclic peptide produced by a fungus,
and many of its suppressive effects are T-cell specific. Cyclosporine
inhibits activation of resting T-lymphocytes, resulting in an inhibition
of IL-2 production. However, once the T-lymphocytes have been
activated, cyclosporine is not effective in suppressing the immune
response. Cyclosporine is absorbed slowly and incompletely from the
GI tract, and excretion is primarily through bile. There is a well
documented enterohepatic cycle. Adverse effects of cyclosporine
include hirsutism, neurotoxicities, hyperkalemia, nephrotoxicity and
hepatotoxicity. The most frequent toxic effects are nephrotoxicity,
hypertension and tremors.

Schwartz 6th Edition, p. 398


www.Xerous.com
Surgery Online

3/3

4. Surgical Complications Associated with the Repair of an Indirect


Inguinal Hernia
The surgical complications associated with the repair of an indirect
inguinal hernia include hemorrhage associated with damage to the
pubic branch of the obturator artery, the deep circumflex iliac vessels,
inferior deep epigastric vessels, the cremaster artery, or external iliac
vessels. Adequate exposure and ligation of the vessels is paramount to
resolving the bleeding problem. Inadvertent suture through either the
iliac vein or artery should be identified and removed. Gentle pressure
to the perforated vessel will usually control the bleeding.
Transsection of the spermatic cord is followed by tenderness and
swelling of the testes in two-thirds of the patients. Testicular atrophy
will occur in approximately one-third of the patients with a transsected
spermatic cord. Routine orchiectomy is unnecessary.
Severance of the ilioinguinal and iliohypogastric nerves in both the
genital and femoral branches of the genitofemoral nerve can occur
during inguinal hernia repair. Fortunately, there are multiple cross
connections between peripheral nerves in the groin. Thus, prolonged
anesthesia of the skin generally does not follow injury to one of these
nerves. Genital femoral neuralgia is a syndrome characterized by
chronic pain and paresthesias in the region of the genitofemoral nerve.
The genital branch of this nerve provides motor innervation to the
cremaster muscle, and sensory innervation to the skin of the penis and
scrotum. The femoral branch of the genitofemoral nerve is sensory to
the skin of the proximal medial thigh.
Other complications include severance of the vas deferens, which
should be repaired if divided.
Damage to the intestines can occur during high ligation of the sac of
an indirect inguinal hernia. Blind suturing is never acceptable.
Similarly, the bowel is commonly involved in a sliding hernia. Great
care must be taken to open the hernia sac on the anterior lateral
aspects, in that bowel is frequently involved in the hernia sac on the
posterior and medial aspects. When injury to the bowel is suspected,
an abdominal incision may be necessary to have full exposure of the
traumatized area to insure that adequate blood supply is maintained,
and that a careful closure of the perforation is performed. Appropriate
antibiotic coverage should be initiated at the time of injury.
Bladder injury can occur on repair of direct inguinal hernias. Defects in
the bladder wall should be closed in two layers, a catheter drainage
established, and repair of the hernia completed.
Femoral vein constriction can occur in a "Cooper ligament repair". The
incidence of this problem is unknown, and recognition is dependent
www.Xerous.com
Surgery Online

4/4
upon the identification of early postoperative deep venous thrombosis,
or pulmonary embolism. The venogram should be obtained and
consideration given to reoperation for removal of the offending
sutures.
Postoperative complications include: urinary retention, scrotal
achromatosis, testicular atrophy, wound infection, missed hernia, and
recurrences.
Reported recurrence rates for indirect inguinal hernias ranges from 1-
7%, whereas direct hernia recurrence occur in 4-10% of the cases.

Nyhus Condon; Hernia

5. Diagnosis of Antithrombin III Deficiency


The coagulation cascade and fibrinolytic systems are
modulated/controlled by many factors. A defect or deficiency of any of
a number of these important proteins can have major clinical
significance.
Antithrombin III is the most important circulating inhibitor of
coagulation. It inhibits clotting factors: activated factor IX(IXa),
activated factor X(Xa), and thrombin. Antithrombin III activity is
increased by binding the heparin. A congenital absence or deficiency of
AT-III predisposes one to arterial and venous thrombosis.
Antithrombin III deficiency is inherited as an autosomal dominant
trait. Most affected individuals are heterozygous for this deficiency,
which occurs in 1:2000 individuals. Heterozygous individuals will have
a 50% reduction in protein concentration or a mixture of mutant and
normal molecules. Laboratory screening involves assessing both AT-III
concentration by immunoassay and the plasma antithrombin and
heparin cofactor activity with functional assays. Patients who develop
acute thrombosis should be treated with IV heparin initially since there
is usually adequate amounts of normal AT-III to complex with heparin.
They subsequently should be treated with oral anticoagulants for life.
Asymptomatic individuals require no chronic anticoagulation but
should receive prophylactic anticoagulation with heparin or plasma
infusions to raise AT-III levels prior to surgical procedures, which may
increase their risk of thrombosis.

Sabiston, Textbook of Surgery, 14th Ed. pp 87-88


Harrison, Principles of Internal Medicine, 13th Ed. pp 1808-1810.

6. Diagnosis of Incarcerated Groin Hernia


www.Xerous.com
Surgery Online

5/5
Incarceration refers to a hernia that is not reducible. Diagnosis of an
incarcerated hernia is usually based on physical exam and history. The
patient should be examined while standing with attention to the groin
area for evidence of a bulge while the patient coughs or tenses the
abdominal musculature. Following this, a gloved finger should be
inserted in the external ring for evidence of the hernia, also while
increased intraabdominal pressure is elicited. The preceding steps
should also be repeated while the patient is supine. Incarcerated
hernias are difficult to differentiate from strangulated ones, and are
therefore considered surgical emergencies.

Sabiston, Textbook of Surgery, 14th ed.

7. Diagnosis of Lidocaine Toxicity


Lidocaine overdosage produces death from either ventricular
fibrillation or cardiac arrest. Less severe side effects include
sleepiness, dizziness, paresthesias, altered mental status, coma,
seizures. At higher concentrations of Lidocaine (toxicity) individuals
will experience decreased hearing, disorientation, muscle twitching,
convulsions, or respiratory arrest.

Goodman & Gilman: The Pharmacological Basis of Therapeutics, pp


320, 861.

8. Diagnosis of Primary Lung Cancer


The diagnosis of lung cancer begins with a thorough history and
physical exam.
The major risk factors are smoking and asbestos exposure (in
smokers).
Clinically, patients may present with cough, hemoptysis, chest pain,
dyspnea, pleural effusion, clubbing, hoarseness, or remain
asymptomatic. Diagnostic studies should begin with chest x-rays.
Suspicious lesions should be followed up with a CT scan to assess local
spread and to check for metastases. The adrenal glands and liver
should be included in the thoracic CT. Histological diagnosis should be
attempted by sputum cytology, bronchoscopy, bronchial biopsy,
brushings or washings, or by percutaneous transthoracic needle
biopsy. Mediastinoscopy is helpful in determining hilar involvement in
patients with nodes larger than 1 cm in the hilum as seen on CT scan.
Bone scans and brain CT are indicated studies in symptomatic patients.
The usual signs of inoperability include:
www.Xerous.com
Surgery Online

6/6
Bloody pleural effusion
Horner's syndrome
Vocal cord paralysis
Phrenic nerve paralysis
Superior vena cava syndrome
Distant mets

Sabiston, 14th ed., page 1750


Schwartz, 6th ed., page 740

9. Diagnosis of TPN-Associated Hypoglycemia


TPN-associated hypoglycemia is caused by sudden slowing of the
glucose infusion, and is the most common cause of hypoglycemia other
than excessive insulin administration. Blood glucose levels below 50
mg/dL are associated with symptoms, which may include mild
confusion, somnolence, obtundation, coma, seizures, diaphoresis,
hypotension, arrhythmias and cardiac arrest. In general, patients who
are undergoing surgery while receiving TPN should have the rate of
infusion reduced to 50 ml/hr. This rate provides an adequate glucose
load to prevent hypoglycemia and is not excessive enough to pose a
risk of hyperosmolar nonketotic coma. When TPN running at a high
rate and is discontinued suddenly, a solution of 10% dextrose in water
should be hung in the interim. Weaning from TPN can be stopped when
the infusion rates are 50 ml/hr or less.

Rombeau, Clinical Nutrition, Parenteral Nutrition, 2nd Edition, W.B.


Saunders Co., 1993.
Sabiston, Textbook of Surgery, 14th Edition, W.B. Saunders Co., 1991.
Schwartz, 6th Ed, p 482

10. Etiology of Post-Op Wound Infections


The occurrence of post operative wound infections are influenced by a
number of factors:
1. Source of bacteria, type of bacteria, bacterial virulence, bacterial
antibiotic resistance, size of bacterial inoculum. 2. Skin preparation
3. Duration of operation >2 hours
4. Extent of tissue damage including inappropriate use of
electrocautery
5. Presence of hematoma or seroma or foreign body
6. Patient age
7. Presence of chronic illness (e.g. renal failure, liver failure, chronic
www.Xerous.com
Surgery Online

7/7
obstructive pulmonary disease, malignancy, diabetes mellitus)
8. Hypotension or hypoxemia
9. Malnutrition
10. Use of immunosuppressive drugs, corticosteroids or chemotherapy
The most common organisms that cause wound infections in non-GI or
biliary tract surgery are Staph and Strep. In GI and biliary tract
surgery it is gram negative and anaerobic organisms that are
responsible.

Schwartz, Principles of Surgery, 1994, p.458

11. Fluid Requirements in Burn Wounds


Expeditious and appropriate fluid resuscitation in burn injury is of
utmost importance. Burn shock will rapidly ensue secondary to
extreme losses in intravascular volume unless these losses are
replaced. Fluid resuscitation will greatly enhance edema but results in
no permanent detriment. Edema in the upper airway or in
circumferential thoracic eschar can, however, prove life-threatening.
Burn fluid closely resembles plasma and, therefore, lactated Ringer's
solution is an excellent resuscitative fluid, being slightly hypotonic and
therefore providing some free water as well. It is generally felt that
colloid administration is ineffective or even detrimental during the first
24 hours secondary to leakage from the intravascular spaces (leading
to pulmonary edema, etc.) During the second 24 hours, colloid exerts a
positive effect upon the plasma volume as it is retained within this
compartment. There is also some theoretical advantage to giving
hypertonic saline, allowing one to initially resuscitate the patient with
less fluid, an important consideration where extensive edema
formation is especially troublesome (see above). The administration of
hypertonic saline is limited by hypernatremia, and it has been shown
that at the end of 72 hrs, total fluids and sodium load are similar
between hypertonic and isotonic solutions.
There are several formulas for estimating the fluid requirements in the
burn victim, the Parkland formula, being the most popular. Isotonic
fluid is given according to the following formula: 4 mL/kg/%TBSA
burned. Half is given in the first 8 hours post burn-not post
presentation- and the second half is given in the next 16 hours. All
formulas are merely estimates and are to be altered early according to
the patient's response to the fluid resuscitation. Bolus therapy is to be
avoided as this results in a net increase in total fluid need (the rate of
edema formation is directly proportional to the rate of crystalloid
administration).
www.Xerous.com
Surgery Online

8/8
Early administration of colloid may be advantageous if hemodynamic
instability occurs or persists. Major indicators of effective intravascular
volume resuscitation are: urine output, PCWP, cardiac output, absence
of significant acidosis, and systemic blood pressure.
Urinary output is the most reliable index as the kidneys are the most
poorly perfused organ in the burn patient and therefore, adequate
renal perfusion implies adequate blood flow to other organs. Urine
flow should be at least 0.5 mL/kg in an adult and 1.0 mL/kg in a child
up to 30 kg. Clinical judgement is of paramount importance in the
determination of adequacy of resuscitation. Electrical burns with
myoglobinuria require urinary outputs of 100-150 mL/hr with the
possible addition of sodium bicarbonate (urinary alkalosis with a pH
>5.6 increases solubility of myoglobin) and mannitol (osmotic
diuretic). During the second 24 hours, dextrose/free water (D5W) and
colloid (0.5mL/kg?% burn, usually as 5% albumin in NS) are added.
Evaporative losses, which are essentially sodium free, become
significant at this time.

Demling RH. Burns. In: Greenfield et al Surgery: Scientific Principles &


Practice. Philadelphia. JB Lippincott, 1993:374.
Goodwin CW, Finkelstein JL, & Madden MR. Burns. In: Schwartz,
Shires, Spencer eds, Principles of Surgery, 6th Edition, NY, McGraw-
Hill, 1994:2343-238.

12. Increased Wound Tensile Strength


There are three phases in wound healing:
1. Inflammatory Phase: involves mainly PMN's and macrophages, this
phase lasts until the wound surface is closed by ectodermal elements
(4-7 days)
2. Proliferative Phase: characterized by the production of collagen in
the wound. The main cell producing collagen is the fibroblast.
Hydroxyprolene and hydroxylysine are found only in collagen and
require hydroxylation to form effective fibrils. The fibrils are held
together by crosslinks. Production of collagen begins about day 7 after
wounding, and continues until approximately day 60, with a rapid
spurt of production occurring from 18 to 50. The increased
proliferation of collagen is reflected in the rapid gain in tensile
strength during this period.
3. Maturation Phase: characterized by the maturation of collagen by
intermolecular cross linking. The wound scar appears to flatten out
and become more pale, taking approximately 9 months. No net
www.Xerous.com
Surgery Online

9/9
collagen production occurs during this phase, only the remodeling of
existing collagen.

Essentials of General Surgery, 2nd Edition pg 120

13. Preoperative Treatment of ITP


High-dose intravenous gamma-globulin is very effective in achieving
an increase in the platelet count preoperatively in patients who do not
respond to steroids or are not candidates for steroid therapy. It is
postulated that intravenous gamma-globulin therapy promotes a rise
in the platelet count because it temporarily reduces platelet
destruction by saturating macrophage Fc receptors, thus producing a
transient blockade of the reticuloendothelial system. A significantly
improved platelet count (100,000-250,000 mm3) occurs within 4-6
days and provides a "therapeutic window" for performance of
splenectomy. In addition to effecting an increase in the platelet counts
of patients who fail to respond to steroids or who are not candidates
for steroid therapy, high-dose gamma globulin therapy is appropriate
for the patient with ITP needing urgent splenectomy in whom a trial of
steroids is not warranted, and in the pregnant patient with ITP late in
the third trimester of pregnancy. Immunizations with polyvalent
pneumococcal vaccine (Pnu-Imune 23 or Pneumovax 23), H.influenzae
vaccine, and Neisseria meningitidis vaccine should be administered as
soon as it becomes likely that splenectomy will be performed. Ideally,
these immunizations should be administered 10-14 days
preoperatively. It is probable that patients receiving steroids will have
a suboptimal early response to these vaccines, but ultimately will
develop protective antibody titers. The availability of blood and
platelets for transfusion should be ensured, although the need for
blood transfusion is rare. Intraoperative thrombocytopenic bleeding
usually ceases after the splenic artery is ligated. In Schwartz's series,
platelets were not used preoperatively and were administered
intraoperatively or postoperatively in only 9 of 120 patients who had
splenectomy for ITP. Although a nasogastric tube is advisable for
postoperative gastric decompression in most patients undergoing
splenectomy, one of the authors (G.F.S.) avoids use of a nasogastric
tube in patients with ITP because of the potential risk of precipitating
hemorrhage from the nose or nasopharynx.

Sabiston, Textbook of Surgery, 14th Ed., pg 1117.


www.Xerous.com
Surgery Online

10/10

14. Prognosis in CA Survival


A growing body of evidence suggests that tumors are immunogenic,
and trigger a T-cell mediated response in the host. As an independent
variable, tumor infiltration with lymphocytes is associated with a
better prognosis than tumors without infiltration; this is especially true
in melanoma and breast carcinoma. This improved prognosis is the
impetus behind various therapies designed to augment the immune
response in these patients, such as interleukin and interferon
therapies.

O'Leary JP, The Physiologic Basis of Surgery, Williams & Wilkins, 1993,
p. 156.

15. Treatment of Burn Wounds


Wound Management: Following airway management and the
institution of resuscitation, care of the burn wound begins.
Initial debridement is best carried out in the tank room or burn
operating room where provisions are present to reduce heat loss and
prevent major electrolyte/fluid shifts. Although bullae may be left
intact in outpatients, all bullae in hospitalized patients should be
debrided. Gentle scrubbing is effective in removing loose necrotic
tissue. Debridement is followed with a chlorhexidine bath. The burn
wound is then covered with an antimicrobial cream, i.e. silver
sulfadine. The unhealed burn wound should be debrided, cleansed, and
antimicrobial cream applied twice daily. Careful monitoring for
invasive wound infection is essential.
Burn wounds in children, in those with 30% total body surface area
(TBSA) and patients with associated multiple system failure are at high
risk for infection. Wound biopsies should be performed freely with one
half of the specimen examined by frozen section for bacteria and the
other processed for quantitative culture and sensitivity. Antibiotic
treatment should be based on sensitivities of organisms to a panel of
antibiotics. Excision of the burn wound in full thickness and deep
partial thickness wounds can begin as early as 3-5 days post burn.
Tangential excisions using a dermatome are usually restricted to 20%
TBSA or two hours of operating time due to blood loss. Primary wound
coverage with autograft, meshed autograft, or homograft should
follow. The donor sites (wounds) must be managed carefully to
prevent infection and promote healing.
Early skin grafting should be considered for facial burns, hand burns,
and circumferential burns of the neck or thorax.
www.Xerous.com
Surgery Online

11/11
Schwartz, Principles of Surgery, 6th Edition, pp 244-250

16. Treatment of Complications of Immunosuppression


The clinical use of immunosuppressive agents has made
transplantation possible. Single agents were used initially, but were
fraught with high morbidity and mortality due to the agent, i.e. total
body irradiation. Multimodal therapy using a combination of agents
has been found to allow maximum immunosuppression with minimum
side effects due to lower dosages of each agent. Specific agents, side
effects and treatment of complications are listed.
Purine Antagonist
Azathioprine - Bone marrow suppression with leukopenia, liver toxicity
- reversible, stop drug
Folic Acid Antagonists
Aminopterin, Methotrexate - Bone marrow suppression - Folinic acid
Alkylating Agents
Nitrogen mustard, Busulfan cyclophosphamide - Alopecia, GI toxicity,
anemia - Reduce or stop drug
Specific Toxicity: Fluid retention, hemorrhagic cystis, cardiac toxicity
Macrolide Antibiotic
Cyclosporine - Hirsutism, neurotoxicity, hyperkalemia, hepatotoxicity
Nephrotoxicity, tremors and hypertension are most common - Reduce
or stop drug
FK506 - Anorexia and weight loss, nephrotoxicity
Corticosteroid
Prednisone - Cushinoid, hypertension, diabetes, peptic ulcer,
personality change, osteoporosis, aseptic necrosis cataracts - Reduce
or stop drug
Antilymphocyte Globulin
ALG - Anemia, thrombocytopenia, allergic reactions - Self limiting
Radiation - Radiation sickness, death - Restrict dosage

Schwartz, 6th Edition, p. 395-402

17. Treatment of Contaminated Soft Tissue Wounds


Contaminated wounds must be differentiated from infected wounds.
Contamination is simply the presence of bacteria in a wound; infection
is the presence of bacteria to an extent that local defenses are
overwhelmed. Generally, proliferation of bacteria beyond 100,000
changes contamination to infection.
Care of the contaminated soft tissue wound entails strict adherence to
www.Xerous.com
Surgery Online

12/12
the principles of good surgical technique:
1) Thorough debridement of devitalized tissue and foreign bodies
2) Precise hemostasis
3) Approximation of tissues to prevent dead space when appropriate
4) Meticulous care to maintain a sterile environment
The contaminated area must be cleansed thoroughly with a bland soap
or antibacterial solution. This can be done under local or general
anesthesia. Embedded materials in skin may require more vigorous
scrubbing with a brush. Foreign bodies should be carefully removed
with forceps. Any tissue with questionable viability is then excised
with sharp debridement. Jagged wound edges are revised to leave no
tissue containing embedded material and to provide more adequate
closure. After thorough debridement, hemostasis should be established
by ligation of bleeding points. The wound is then copiously irrigated
with normal saline. There have been no studies showing any
advantage in using antibacterial irrigants. However, pulse irrigation
under pressure has been shown to decrease the infection rate
considerably. If the soft tissue appears to be completely
decontaminated, the wound may be closed in layers. Otherwise, it is
best to leave the subcutaneous tissue open with closure only to the
level of the fascia. Saline soaked gauze is used to pack the wound
lightly. In 4-7 days, the wound can be closed utilizing meticulous,
strict sterile techniques. Even with the most careful of techniques,
infection rates about 15% can be expected.

Nora, Paul F: Principles of Operative Surgery, 3rd ed. WB Saunders,


1990, pp 9-11.
O'Leary, JP: Physiologic Basis of Surgery, W&W, 1993, pp 103-104.

18. Treatment of Factor XI Deficiency


Factor XI, plasma thromboplastin antecedent (PTA) deficiency, or
Rosenthal's syndrome, is a mild disorder occurring mainly in patients
of Jewish ancestry. These patients may undergo major procedures
without significant bleeding. Factor XI or PTA will promote hemostasis
with only 10% of the normal level available. The factor is stable in
stored blood, and has a half-life of 40-80 hours. The deficiency state is
an inherited autosomal dominant and can be corrected with the
administration of FFP.

Schwartz, 6th Edition, pp 102, 114


www.Xerous.com
Surgery Online

13/13

19. Treatment of Syndrome of Inappropriate Secretion of ADH


SIADH refers to the "syndrome of inappropriate antidiuretic hormone
secretion" and is the term used to describe the excessive secretion of
vasopressin beyond that needed to promote homeostasis. The
persistent secretion of vasopressin produces a low urinary output with
high osmolarity and a profound dilutional hyponatremia (isovolemic
hypotonic hyponatremia). This syndrome occurs most frequently
following a closed head injury but may also occur from ectopic
secretion from oat-cell carcinomas of the lung.
Diagnosis:
1) hyponatremia with hypoosmolarity
2) hypertonic (urine>plasma) even after free water administration
3) inappropriately high urinary Na after water loading
4) normal renal, adrenal and thyroid function
5) no clinical evidence of volume depletion or overload
6) the absence of diuretic administration
Treatment:
Restriction of free water intake to less than 1000 mL per day is
appropriate initial therapy. In marked (Na <110 mEq/l) or
symptomatic hyponatremia, hypertonic saline may be cautiously
administered, often in conjunction with Lasix. The serum Na should
not be corrected more than 1 mEq/L/hr during the first 24 hours of
therapy because of the possibility of neurologic complications,
including seizures, central pontine myelinolysis, paresis and coma.

Schwartz 5th edition pg.24


Cameron: Current Surgical Therapy 4th ed, pg. 1006

20. Action of Hypoxia on Wound Healing


The Bottom Line: Hypoxia has an adverse effect on the wound healing
process.
Specifically: Oxygen is essential for the hydroxylation of proline
molecules in the synthesis of collagen. A lack of oxygen results in
failure of this step. This leads to an unstable collagen molecule which
is rapidly degraded. Therefore, the wound healing process is defeated
leading to insufficient wound strength.

Schwartz, Principles of General Surgery, 6th ed, p. 285

21. Activation of CD-4 Lymphocytes


www.Xerous.com
Surgery Online

14/14
With the use of monoclonal antibodies it has been possible to define
several cell-surface markers on developing and mature T-cells. Some,
such as CD-3 and CD-2, are present on all peripheral T-cells, whereas
others define functionally distinct subsets. CD-4 molecules are present
on approximately 60% of mature peripheral T-cells, and some
monocytes. They serve as a marker for T-helper-inducer cells. CD4+
helper-inducer cells provide help in the generation of cytotoxic T-cells
and antibody-secreting B-cells. The delayed hypersensitivity reaction is
mediated by CD4+ cells. CD4 molecules bind to Class II molecules of
the MHC. Macrophages process and present antigens in conjunction
with Class II histocompatibility antigens to CD4+ T cells for induction
of cell-mediated immunity. This results in specifically sensitized T
lymphocytes called memory T cells, which remain in circulation for a
long time, often several years. When the individual is reexposed to the
specific antigen (eg. tuberculin), memory T lymphocytes are
stimulated to divide and release a variety of lymphokines.
Lymphokines amplify the response by recruiting inflammatory cells,
activating them and keeping them at the site. The most important
lymphokines appear to be macrophage chemotactic factors,
macrophage inhibitory factors, IL-1, TNF, and IFN. Only a very small
percentage of the mononuclear cell infiltrate is made up of memory T
cells in a fully developed delayed hypersensitivity reaction. In T-cell
mediated graft rejection, Class II antigens are presented to and
recognized by CD4+ helper cells. This leads to release of IL-1 from the
antigen presenting cell, promoting the proliferation of CD4+ cells and
the release of IL-2 from the CD4+ cells. IL-2 further augments CD4+
cell proliferation and, along with IL-4 and IL-5, provides helper signals
for B cell differentiation.

Mandell, Douglas, Bennett. Principles of Infectious Diseases, 3rd ed.,


1990, pgs. 104-105

22. Acute Rejection of Renal Transplant


Acute renal transplant rejection is a delayed type hypersensitivity
process occurring after 3-5 days with the peak incidence at about five
weeks. The process is usually reversible and occurs in 65-85% of renal
allografts. Clinical features include graft swelling and tenderness,
fever, oliguria, and hypertension. Diagnosis is confirmed by an
increased serum creatinine. Biopsy will show a prominent interstitial
and perivascular mononuclear infiltrates with interstitial edema.
Macrophages and T lymphocytes are predominant. In severe rejection,
some polymorphonuclear leukocytes are present. Acute rejection
www.Xerous.com
Surgery Online

15/15
episodes may be precipitated by viral syndromes, direct damage,
crossreacting antigens, or augmented allograft/host immunologic
status.

Miller, pgs. 94-95

23. Agent with Greatest Catabolic Effect


The response to injury is characterized by an initial "catabolic phase"
or adrenergic-corticoid phase, since it corresponds to high levels of
these agents following injury. Glucose is the primary metabolic fuel
source in contrast to lipids, which are the major source of energy
during starvation. Proteins are mobilized in large quantities for
gluconeogenesis and to provide aminoacids needed for "acute phase
proteins" and wound repair. Of the hormones involved in the "catabolic
phase" (catacholamines, glucagon, glucocorticoids and low
concentrations of insulin). The glucocorticoids drive the catabolic
effects while the others mobilize glucose (glucagon) and increase the
metabolic rate (catacholamines). Interleukin 1 (IL-1) and Tumor
Necrosis Factor (TNF) also called cachexin are participants in the
catabolic response.
However, glucocorticoids should be considered the agent with the
greatest catabolic effect.

Schwartz, 6th Ed., pg 82

24. Bacteria Producing Most Potent Exotoxins


Although a number of gram-positive and gram-negative bacteria can
produce symptoms related to exotoxins, the most serious exotoxins
result from the gram-positive rod-shaped micro-organisms of the
Clostridia group. These organisms, which are obligate anaerobes, are
widely present in the soil and intestinal tracts of humans and animals.
Clostridia perfringens is the clinically most important organism and
produces a number of potent exotoxins: - alpha toxin - a lethal,
necrotizing, hemolytic lecithinase - theta toxin - some strains produce
this hemolysin - kappa toxin - a collagenase - mu toxin - a
hyaluronidase - nu toxin - a deoxyribonuclease Clostridia perfringens
also produces an endotoxin. Wound infections produced by this
organism can vary in severity from simple contamination to Clostridial
cellulitis and Clostridial myonecrosis. Often a combination of multiple
species of Clostridium is present. Clostridial cellulitis usually carries
mild systemic effects with little or no toxemia. However, it may spread
www.Xerous.com
Surgery Online

16/16
rapidly and require radical surgical intervention. Clostridial
myonecrosis or gas gangrene with severe progressive destruction of
muscle and fascia requires aggressive surgical and medical treatment.
Clostridia difficile and its exotoxin is the most important cause of
antibiotic-associated pseudomembranous colitis. The diarrhea
produced by this condition will sometimes subside when the offending
antibiotic is stopped. In more severe cases, oral Metronidazole is the
treatment of choice. Oral Vancomycin is also effective and in mild
cases, cholestyramine, an ion exchange resin that binds the toxins,
may also be effective.
Clostridia tetani is a non-invasive bacterium, which produces a potent
neurotoxin, which may begin producing symptoms after a variable
time. Muscular rigidity and spasm, sometimes leading to exhaustion
and asphyxiation in severe cases can lead to death. Progressive
debridement of wounds and treatment with Penicillin as well as
aggressive, supportive care are the mainstay of treatment.
Clostridia botulinum, a food contaminant, produces a potent
neuroexotoxin, which causes gastrointestinal symptoms followed by
diplopia, blurred vision and dysphagia leading to paralysis. Supportive
care may require assisted ventilation.

Schwartz, 6th Ed., pp 158

25. Cancer Associated with Vinyl Chloride Exposure


Vinyl chloride is a known carcinogen and is known to cause cancer
specifically in the lung, liver and brain. It is a chemical used in the
plastics industry. Industrial exposure to vinyl chloride has been linked
to the development angiosarcoma of the liver.

DeVita et al:, Cancer: Principles & Practice of Oncology, 4th Edit. 1993,
pp 205.
Robbins, Pathologic Basis of Disease, 4th Edit.

26. Characteristics of Adoptive Immunotherapy


Adoptive immunotherapy is the transfer of donor immune lymphoid
cells to a recipient to mediate tumor destruction. In man, autologous
lymphocytes obtained through leukophoresis are generated into
lymphokine activated killer (LAK) cells by interleukin-2. These
cytotoxic lymphocytes have demonstrated objective responses against
bulky metastatic cancers in some patients. Toxicity of interleukin-2
includes fluid retention and renal dysfunction. This immune system
www.Xerous.com
Surgery Online

17/17
manipulation may provide useful adjuvant therapy in the treatment of
sarcomas, lymphomas, leukemias, melanoma and breast carcinoma.

Schwartz, Principles of Surgery, 6th ed. p. 368


Rosenberg, SA Adoptive Immunotherapy of Cancer: Accomplishments
and Prospects, Cancer Treat Rep 68:233, 1984.

27. Characteristics of Cytokines


Cytokines are a group of soluble hormone-like glycoproteins that
function as immune cell regulators. These potent agents generally act
locally to both modify cell-specific behavior and to remodel the
extracellular architecture. Interactions are complex, and single agents
may exhibit a plethora of activities with different cell types and matrix
proteins. Specific classes of cytokines include, but are not limited to,
the growth factors, tumor necrosis factors, interleukins, and
interferons. Known cytokines include IL-1 through IL-10, Interferon
gamma, GM-CSF, and TNF. The principal cytokines studied for their role
in the acute phase response are IL-1 and TNF. IL-1 may be the
underlying instigator of the acute phase response. TNF may have a role
in the development and/or propagation of the cachexia.

O'Leary, The Physiologic Basis of Surgery, Williams and Wilkins, 1993


Schwartz, 6th Ed, p 389

28. Characteristics of Delayed Wound Healing


In contradistinction to primary closure where disrupted tissue is
approximated by sutures, staple or tapes, and wound healing
progresses unimpaired through synthesis, deposition and crosslinkage
of collagen to provide optimal strength and integrity, the delayed
primary closure waits with the approximation of the wound for several
days. This technique and wound management has been a mainstay of
battlefield surgery, where wound debridements of multiple fragment
wounds are conducted under sterile conditions with local anesthetic.
The wounds are then packed with sterile gauze, and the patient is
returned to the operating room 3-5 days later. At this dressing change
wound is examined carefully for evidence of infection or additional
devitalized tissue, which was not removed at the initial operation. If
the wound has clean granulations and appears without infection the
wounds are closed primarily. This technique has been used effectively
in civilian practice. However, the dressings are changed daily or twice
daily basis using sterile technique and sterile dressing.
www.Xerous.com
Surgery Online

18/18
Use of peroxides and iodophores in these open wounds is to be
avoided in that these solutions destroy host tissues as well as bacteria.
Angiogenesis proceeds to provide enhanced blood supply and needed
oxygen. Leukocytes are attracted to destroy and remove bacteria, and
the phases of healing progress to improve the wound environment for
soft tissue closure. Wound tensile strength following delayed primary
closure becomes the same as the primarily closed wound with time.
Spontaneous closure or "secondary" closure occurs when the margins
of the open wound move together by the biologic process of
contraction.

Sabiston, 6th Edition, pp 280

29. Conditions Associated with Early Sepsis


Septic shock, that is thought to be due to progressive systemic
infection is initially manifested by the development of a
hypermetabolic state which leads to increased heat production and
subsequent diversion of blood flow to the skin through the opening of
cutaneous arteriovenous shunts to allow for heat loss. The capacitance
of the cutaneous vascular bed expands depriving other areas of the
body from blood flow unless simultaneous repletion of intravascular
volume is carried out. Further, an intense intravascular inflammatory
reaction is activated primarily by endotoxin leading to a diffuse
increase in microvascular permeability, which also leads to
hypovolemia. Consequently, fever, low urine output, hypotension with
flat neck veins, and warm pink skin due to decreased systemic vascular
resistance are seen initially. The cardiac output and mixed venous
oxygen content are both increased while the oxygen consumption is
decreased. This condition is known as early sepsis or "red shock".

Sabiston, Textbook of Surgery, 14th ed., pg 38, 41

30. Characteristics of Enterocytes


Enterocytes, or absorptive cells, are found in the mucosa of the small
and large bowel. The normal enterocyte lives for a little more than 2
days. They are columnar and are the principal cells of the villus.
Enterocytes absorb a variety of nutrients including Ca, Fe and H2O.
The primary fuel source of enterocytes is glutamine.

Principles and Practice of Surgical Pathology, 2nd Edition, Vol II, pgs
1121-23
www.Xerous.com
Surgery Online

19/19

31. Characteristics of Fibronectin in a Healing Wound


The extracellular matrix, which supports cells consists of collagens,
non-collagenous glycoproteins, proteoglycans and elastin. Fibronectin
is the best-characterized of the noncollagenous glycoproteins. It is a
dimeric protein containing separate domains for binding collagen,
heparin and fibrin, as well as specific cell-surface receptors.
Fibronectin is produced by fibroblasts and several other cells in both a
soluble (serum) and insoluble (cellular) form. Fibronectin, like fibrin,
appears in a healing wound during the inflammatory phase; it serves
to facilitate the attachment of incoming fibroblasts to the wound's
fibrin latticework.

O'Leary JP, The Physiologic Basis of Surgery, Williams & Wilkins, 1993,
pp 33, 96

32. Characteristics of Flouroquinolone Antibiotics


The flouroquinolones - ciprofloxacin, norfloxacin, enoxacin, and
ofloxacin, have a wide spectrum of activity, may be used orally or
parenterally, and have minimal adverse side effects. They are effective
against most isolates of Enterobacteriaceae, are highly active against
H. Influenzae, N. gonorrhoeae, B. catarrhalls, and P. aeruginosa,
staphylococci including methicillin-resistant strains, as well as Brucella
species, Legionella species, C. trachomatis, M. pneumoniae and M.
tuberculosis. Organisms that they may not be active against include
Streptococci and anaerobic cocci and bacilli. The quinolones have a
wide volume of distribution and good cellular penetration. Also, they
do not alter the anaerobic intestinal flora or select for plasmid-
resistant strains.
The quinolones are bactericidal and their mechanism of action is
interference with the enzyme DNA gyrase, needed for bacterial DNA
replication.
Adverse effects are limited to increased theophylline levels, and an
increased anticoagulant effect, thought to be secondary to an
alteration of hepatic metabolism.

Care of the Surgical Patient, Scientific American, Vol. IX, Ch. 3

33. Characteristics of Glucocorticoid Receptors


Glucocorticoid (steroid) receptors are linear peptides (MW ð 87,000) in
the cytosol of target cells. Almost every type of cell found in the body
www.Xerous.com
Surgery Online

20/20
has some glucocorticoreceptors. These receptors bind to cortisol,
prednisone, and dexamethasone, but do not bind to nonglucocorticoid
steroids. The strength of the response is related to the concentration
of the steroid available to the cells. The receptor undergoes a
conformational change when complexed with a steroid. This activated
complex is translocated to the nucleus where it attaches to chromatin
and activates DNA/mRNA replication.

Miller, Physiologic Basis of Modern Surgical Care, page 936

34. Characteristics of Growth Factor Receptors


Growth factors are polypeptides that modulate cellular function and
regulate cellular growth. The specific response depends more on the
type of the growth factor receptor than the specific growth factor
itself. Growth factor receptors are inactive until bound. When bound to
ligand, activation of the intrinsic enzymatic activity of the receptor
occurs and signal transduction follows.
Receptors have been classified into three major types.
Type I-bind epidermal growth factor (EGF), transforming growth
factor-a, and v-erb B.
Type II-bind insulin and insulin-like growth factors
Type III-bind platelet-derived growth factor (PDGF), fibroblast growth
factor (FGF), and some protooncogenes. Growth factors and oncogenes
are closely associated in oncogenesis, although the mechanism
remains unclear.

O'Leary, The Physiologic Basis of Surgery, Williams & Wilkins, 1993


Schwartz, 6th Ed, p 317

35. Characteristics of High-Dose Fentanyl Analgesia


Fentanyl, an opiate drug, is one of the most common narcotics used in
anesthesia. It is highly soluble lipid and approximately 100 times more
potent than morphine. Narcotic drugs are used for premedication,
analgesia, and anesthesia by IM, IV, oral, epidural, and subarachnoid
routes. Narcotic drugs act via several receptors (mu, kappa, delta,
sigma) although all of their effects cannot be explained by this
mechanism alone. Physiologically, narcotic drugs will diminish pain
sensation, cause pupillary constriction by inhibiting the Edinger-
Westphal nucleus in the brain; cause respiratory depression by altering
brain stem response to CO2 and/or causing sedation; cause
nausea/vomiting by stimulating the chemoreceptor trigger zone in the
www.Xerous.com
Surgery Online

21/21
medulla and cause decreased GI motility. Cardiovascular effects of
narcotics include: moderate peripheral vasodilation and histamine
release, which may cause hypotension; decrease heart rate by
inhibition of central vagal nucleus in the medulla. Fentanyl in large
doses is typically better tolerated than other narcotics because it
causes little or no histamine release and direct cardiodepressive
effects are minimal. At high doses, however, opioids can cause skeletal
muscle rigidity making ventilation difficult without muscle relaxants. It
has been most frequently reported with fentanyl, probably due to its
rapid uptake by skeletal muscle.

Dripps et al, Introduction to Anesthesia, 8th Ed, pp 102-109

36. Characteristics of Life Table Analysis


Essentially, the life table is a device to determine whether there are
differences between treatments in patient survival over time. The
analysis takes into account varying factors, such as patients entering
the study at different times, patients who leave the study before the
end of the study period, and patients who are still survivors at the end
of the study period. The life table analysis is used for nominal variables
(variables which are not directly measured but are most commonly one
of several treatment options). There are a variety of statistics used to
analyze the data including the Kaplan-Meier technique and the Cox
proportional hazard model.

O'Leary, p. 249

37. Characteristics of Bacterial Lipopolysaccharide


Endotoxin, the agent responsible for septic shock, has been identified
as the lipopolysaccharide (LPS) outer membrane common to gram-
negative bacteria. The outer membrane consists of an inner leaflet of
ordinary phospholipids, and the outer leaflet consists of
lipopolysaccharide molecules. This molecule is extremely toxic to
humans and is also known as bacterial endotoxin.
LPS consists of:
1. A toxic lipid A portion: a phospholipid containing glycocyamine
rather than glycerol.
2. A core polysaccharide, containing some unusual CHO molecules and
is fairly constant in structure among related species of bacteria.
3. O-antigen polysaccharide side chains: This being the major surface
antigen of gram negative cells. It is highly variable among species and
www.Xerous.com
Surgery Online

22/22
subspecies and is a major determinant of antigen specificity.
The interaction between LPS and host cells initiates a complex
spectrum of physiologic events. Tumor necrosing factor/cachetin
(TNF), a cytokine purified and sequenced by Beutler and Ceramie plays
a major role in the constellation of septic shock symptoms and findings
along with IL-1.
Other cytokines IL-6, IL-8, GCSF have been identified as contributors
to the events.

Sherris, Medical Microbiology, 2nd Edition, p. 20


Schwartz, 6th Ed, p 141

38. Characteristics of Local Anesthetics


There are two basic types of local anesthetics.
In the ESTER group are cocaine, procaine, 2-chloroprocaine, and
tetracaine.
The AMIDE group consists of lidocaine, bupivocaine, prilocaine,
mepivacaine, and etidocaine.
Structurally, all local anesthetics are tertiary amines; ammonia
molecules with each of the three hydrogen atoms replaced by organic
groups. Essentially, you have a hydrophilic portion and a hydrophobic
portion, connected by an amide or an ester. They temporarily interrupt
function of the sodium channel in the cell membrane or neurons in the
vicinity in which they are injected, thus inhibiting propagation of nerve
impulses and therefore transmission of pain impulses. One difference
between the local anesthetics is their duration of action, which can
vary from 45 minutes to 12 hours or more, and can be potentiated by
such agents as epinephrine, which slows local clearance by causing
vasoconstriction. The important difference between the two classes of
local anesthetics is their potential to cause allergic reactions, which
can vary from rash to full cardiopulmonary arrest. While the amides
are rarely implicated in allergic reactions, all esters are broken down
into para-aminobenzoic acid (PABA), which is highly allergenic in
humans. Toxicity is more likely to occur with injection into a highly
vascular area, with rapid delivery of a bolus of the agent to the
myocardium, which can cause profound myocardial depression. This is
a risk with all local anesthetics and is not an allergic reaction but
rather an overdose. While LD50 curves have been established for all
the different agents, individual responses vary widely.

Care of the Surgical Patient Scientific American, Vol. II, Ch. 5


www.Xerous.com
Surgery Online

23/23

39. Characteristics of Microglia Cells


Microglia are small cells of the nervous system that become mobile and
phagocytic in response to inflammation, phagocytozing necrotic tissue,
microorganisms and foreign substances that involve the CNS.

Seely, Anatomy and Physiology, 1995, p.375

40. Characteristics of Prophylaxis Pneumocystis Carinii Infection


For HIV-infected patients with CD4 T-cell counts of 200/mm3 or more,
cell counts should be monitored at least every 3-6 months. Patients
with counts below 200 should receive P.carinii pneumonia prophylaxis.
Patients with constitutional symptoms, such as thrush or unexplained
fever greater than 37C for more than two weeks should also receive
prophylaxis, regardless of the CD4 count. Prophylaxis should be
continued for the patient's lifetime. Oral TMP-SMX (Bactrim,
trimethoprim-sulfamethoxazole) is recommended for prevention of
P.carinii pneumonia in a dosage of one double-strength tablet daily.
Aerosolized pentamidine, administered by either the Respirgard II
regimen (300 mg/mo) or the Fisoneb nebulizer (loading regimen of
five 60 mg doses over two weeks, followed by a 60 mg dose every two
weeks) is recommended. The use of aerosolized pentamidine has been
associated with a variety of complications, including spontaneous
pneumothoraces and extrapulmonary pneumocytosis. Other agents
that have demonstrated efficacy in preventing P.carinii pneumonia
include Dapsone and Pansidar (pyrimethamine-sufadoxine).

Future Therapies in the Management of Critically Ill AIDS Patients,


Torres et. al. Hematology and Oncology Clinics of North America, 1991,
9(9);1543-8
CDC Release Recommendations for Prophylaxis Against Pneumocystis
Carinii Pneumonia, American Family Physician, 1992, 46(1);284-6

41. Characteristics of Radiation Injury


Organ Acute Changes Chronic Changes
Skin Wet or dry epidermitis Running Radiodermatis Ulceration,
Epilation
GI tract Edema, ulceration, infection, stricture, ulceration, diarrhea,
hepatitis, perforation
Kidney Nephritis, renal insufficiency
Bladder Dysuria Ulceration
www.Xerous.com
Surgery Online

24/24
Gonads Sterility atrophy, menopause
Hemopoietic tissue Lymphopenia, pancytopenia
Bone Cessation of epiphyseal growth, necrosis
Lung Pneumonitis Pulmonary fibrosis
Heart Acute pericarditis, myocarditis Chronic pericarditis myocarditis
Eye Conjunctivitis Cataracts
Nervous system Cerebral edema Radiation myelitis

Radiation enterocolitis: Dose dependent: 4000 rad enterocolitis


uncommon; 5000 rad all will have some degree of damage (5-10% will
be significant). Factors increasing likelihood of radiation injury to
bowel are, advancing age, hypertension, arteriosclerotic vascular
disease, diabetes, and previous surgery with adhesions. Mild acute
changes are due to effects on epithelial cells. Serious and later effects
are due to effects on vessels with development of progressive
vasculitis. Early symptoms, which usually resolve rapidly with
cessation are: nausea, vomiting, diarrhea, cramping, rectal bleeding,
tenesmus. Late symptoms are: atrophic mucosa, ulceration, stricture,
abscess formation, fistula, perforation.
Surgical caveats per Morganstern:
a. Avoid entry into abdomen unless forced to operate
b. Avoid extensive resections and multiple anastomoses; long
segments of stricture should be treated with side to side
stricturoplasty
c. Avoid extensive lysis of adhesions
d. Use frozen sections to guide resections
e. Protect anastomosis, colostomy for large bowel, long tube for small
bowel
f. Prevention with pelvic sling if postoperative pelvic radiation
anticipated

Schwartz, 6th Ed, pp 1211-1241

42. Characteristics of Steady State


A steady state is achieved when the rate of drug elimination is equal to
the rate of administration. No matter what is the method of
administration, it takes 5 halflives of the drug to achieve the steady
state. If a drug's halflife is eight hours, it will take approximately 40
hours to achieve the steady state. This time can be shortened by giving
an initial dose larger than the routine dosage. The actual concentration
of the drug achieved at steady state is dependent on the administered
www.Xerous.com
Surgery Online

25/25
dose, percent of absorption, frequency of administration and rate of
elimination or degradation.

O'Leary, The Physiologic Basis of Surgery, 1st Ed., p. 230

43. Characteristics of Succinylcholine


Succinylcholine is a muscle relaxant whose primary action is to prevent
spontaneous or reflex contraction of striated muscle by blocking the
myoneuronal junction. It belongs to the depolarizing class of
neuromuscular blockers, which bind in non-competitive fashion to
receptor sites on the muscle side of the myoneural junction. These first
initiate muscle contractions, then prevent post-synaptic acetylcholine
stimulation by continuing to occupy the receptor. The advantages
include a very short duration of action, usually less than 5 minutes.
However, it can increase the risk of aspiration and worsen glaucoma
by producing generalized contractions of striated muscle that results in
increased gastric and ocular pressures, and it can produce muscle pain
postoperatively. It also has muscarinic effects, most notably
bradycardia.
Hyperkalemia and serious cardiac dysrhythmia can also occur.
In some patients the clinical syndrome of malignant hyperthermia has
occurred. This syndrome is noted by a striking increase in metabolism
causing intense heat production, carbon dioxide and lactate production
as well as associated respiratory and metabolic acidosis can occur. This
condition is treated supportively by cooling the patient and
administering Dantrolene up to 10 mg/kg IV.

Cerra, pg. 753-754

44. Characteristics of Vitamin K


Vitamin K is present in most edible vegetables, particularly in green
leaves. It is also produced in vivo by intestinal bacteria.
Physiologically, vitamin K is a component of a specialized microsomal
enzyme system that affects the posttranslational gamma carboxylation
of precursor proteins of the clotting cascade. Specifically, factors V,
VII, IX, and X. Approximately 80% of ingested vitamin K is absorbed
from the small bowel into the intestinal lymph. Because it is a fat-
soluble substance, deficiency states can occur in association with fat
malabsorption. In addition, long-term use of oral antibiotics may
temporarily eliminate intestinal bacteria as a source for vitamin K and
promote deficiency when dietary intake is suboptimal.
www.Xerous.com
Surgery Online

26/26
Warfarin derivatives, i.e. coumarin, induce hypoprothrombinemia and
an anticoagulated state by affecting primarily the g-carboxylation of
the precursor protein. This is a competitive inhibition that can be
overcome by administration of supplemental vitamin K.
Newborn infants tend to lack vitamin K stores for several reasons.
These include inherently low stores at birth, an immature gut flora,
and limited dietary intake of the vitamin. Routine determination of
prothrombin time should precede surgical procedures when patients
have minimal dietary intake or prolonged antibiotic therapy and poor
nutrition is present. Subjects with less than 70% of normal activity
should receive corrective therapy with vitamin K. Deficiency states of
vitamin K can be differentiated from hypoprothrombinemic states
induced by liver disease by measurement of the noncarboxylated
prothrombin precursor that accumulates in the vitamin deficiency.

Wilson, Harrison's principles of Internal Medicine, McGraw Hill


Publishing, 1991, p 441.

45. Chemotherapy Causing Hemorrhagic Cystitis


Cyclophosphamide is the agent most commonly associated with
hemorrhagic cystitis and can occur in up to 10% of patients due to
active metabolites including acrolein. The risk can be decreased with
adequate hydration with 3-4 liters of fluid per day and administration
of N-acetylcysteine (MESNA) in an attempt to inactivate the
metabolite.

DeVita, Principles and Practice of Oncology, 4th Ed., p.404

46. Compensation for Isovolemic Anemia


Anemia causes problems when oxygen delivery is inadequate.
Compensation for anemia occurs by increased tissue perfusion through
an increased cardiac output, a decrease in blood viscosity and
decreased peripheral resistance. Oxygenation is also improved with
elevation of 2,3-DPG, which often accompanies anemia, an
autoregulatory function of red cells.

Miller, p. 249
Sabiston, p. 95

47. Testing of B Cell Function


www.Xerous.com
Surgery Online

27/27
In vitro assays are used either to define the presence of specific
components of the immune system or to test the activity of these
components, once identified. B-cell function can be quantitated by
enumerating those lymphocytes with immunoglobulin isotopes on
their surface membrane as well as by measuring serum
immunoglobulin levels. It is possible to measure an antigen-antibody
complex by measuring its binding with protein A of staphylococcus or
the binding of the C1q component of complement. In addition,
blocking of Fc receptors in a reaction such as the antibody-dependent
cell cytotoxicity assay can reflect the presence of antigen-antibody
complexes. Polyethylene glycol precipitation of immune complexes can
be simply defined by differences in light absorption. Complement
components can be accurately measured, and deficiencies involving
the pathway of complement activation cascade can be identified. To
date in vivo assays of immune function are not sufficiently
reproducible or reliable to use in a clinical setting.

Miller, Physiologic Basis of Modern Surgical Care, pp 79, 81

48. Etiology of IgG Mediated Thrombocytopenia


ITP, Idiopathic (immune) Thrombocytopenic Purpura is an acquired
hematologic disorder characterized by subnormal platelet count,
increased megakaryocytes, absence of any systemic disease or history
of drug ingestion that induces thrombocytopenia. The spleen produces
antiplatelet IgG antibodies that form platelet-antibody complexes that
are sequestered in the spleen.
Female to male 3:1.
Sx: Bleeding, petechia, thrombocytopenia.
Rx: Steroids, splenectomy.

Schwartz, Principles of Surgery, 1994, p.1440-1

49. Etiology of Prolonged Bleeding after Aspirin Use


Aspirin can prolong the bleeding time, especially in patients with
bleeding disorders through several mechanisms: 1. It may impair the
reaction of platelet release and the second phase of ADP-epinephrine
induced aggregation.
2. It inhibits cyclooxygenase, which prevents formation of
thromboxane A2. Complete inactivation occurs with an aspirin dose of
160 mg/d.
3. It decreases thrombus formation on exposed subendothelium.
www.Xerous.com
Surgery Online

28/28
Aspirin in a single dose will affect all platelets irreversibly for 7-10
days (the lifetime of a platelet), so it should be discontinued 7-10 days
prior to elective operations in order to restore the bleeding time to
normal.

Williams, Hematology, 3rd Ed.


Goodman & Gilman's, The Pharmacological Basis of Therapeutics, 8th
Ed

50. Fever-Reducing Action of Aspirin


Fever is caused by the effects of endogenous pyrogens on the anterior
hypothalamus and is characterized by a regulated rise in body
temperature. An example is IL-1 causing an increase in the synthesis
of PGE2. PGE2 directly increases the set point of the hypothalamic
temperature center causing fever. Hyperthermia (on the other hand) is
not caused by endogenous pyrogens, is unregulated (temperature
exceeds the thermoregulatory set point), is not defended by
physiologic mechanisms, and does not respond to antipyretics.
Aspirin gains its antipyretic effects by way of its inhibition of the
cyclooxygenase pathway of arachidonic acid metabolism:
(1) LIPOXYGENASE PATHWAY ®Leukotrienes---® HPTE---
PHOSPHOLIPID---Phospholipase---ARACHIDONIC ACID---®
ARACHIDONATE
(2) CYCLOOXYGENASE PATHWAY---Glucocorticoid inhibited here
Inhibited by Aspirin and other NSAIDS---PGG2---PGD2---PGH2---
Prostacyclin PGE2---PGF2---Thromboxane A2
In addition, aspirin is the only cycloxygenase inhibitor which is
irreversible

Buechter KJ, Byers PM. Nutrition and Metabolism. In: O'Leary, PJ ed.
The Physiologic Basis of Surgery, Philadelphia, W&W, 1993:85-86
Wyngaarden JB, Smith LH: Cecil Textbook of Medicine, 17th ed,
Philadelphia, WB Saunders, 1985:p. 1240

51. HLA Matched Kidney Donor/Sibling


Recipient HLA antigens have formed the basis for transplantation
tissue typing for many years. Because of extreme polymorphism, only
rarely do two unrelated individuals share all the antigens expressed.
Relatives, on the other hand, often share some antigens because each
person inherits one set of HLA antigens from each parent and all the
antigens are expressed. There are two classes of HLA antigens, I and
www.Xerous.com
Surgery Online

29/29
II.
Class I antigens are expressed on portions of the MHC supergene
called the HLA-A, HLA-B and HLA-C loci.
Class II antigens are expressions of HLA-D, D, DQ, and DW/DR subloci.
HLA-A, -B, and -D pairs are usually inherited together and the antigens
originating from one chromosome are called the HLA haplotype.
Therefore, there are four possible combinations for any child to inherit.

Schwartz, 5th Edition

52. Impairment of Wound Healing


A. Infection: Generally, proliferation of bacteria beyond 100,000/g of
tissue changes a wound from contamination to infection.
Factors, which may promote infection include foreign bodies,
persistence of necrotic tissue and impaired local defenses. Conditions
which may hinder the host defenses against infection can be systemic
such as: diabetes, AIDS, uremia, carcinoma. Local factors include poor
circulation or radiation injury. Inadequate hemostasis with allowance
of a wound hematoma may also lead to infection by providing bacteria
with serum nutrients and a desirable environment as well as
preventing the influx of the host's immune defenses.
B. Malnutrition: Poor nutrition inhibits proper wound healing via
hypoproteinemia and a deficiency of critical amino acids. For example,
cystine which is essential for collagen synthesis. Vitamin C is also an
essential cofactor in the crosslinking process of collagen synthesis. Yet
another element is zinc, which is important for epithelization, collagen
synthesis and wound strength.
C. Decreased O2 Tension: Although anemia does not appear to impair
wound healing significantly, oxygen tension in the wound itself does.
Oxygen tension depends mostly on local perfusion of the wound.
Elevation of temperature increases perfusion and increases oxygen
tension. Atherosclerosis would also impair healing secondary to
decreased oxygen supply and nutrient delivery.
D. Cancer: Cancer inhibits wound healing through a variety of
mechanisms, i.e. change in taste perception, hypothalamic function,
tryptophan metabolism. These can contribute to anorexia and
therefore to poor nutrition. GI tumors may inhibit absorption of
nutrients. Protein catabolism may be accelerated. Some tumors may
preferentially consume vitamin C, limiting its availability. Lastly,
cancer patients may be relatively anergic, resulting in abnormal
inflammatory cell activity.
E. Steroids/Immunosuppression: Adrenocortical steroids inhibit all
www.Xerous.com
Surgery Online

30/30
phases of wound healing. They impede epithelization, contraction, and
macrophage migration. Collagen accumulation and angiogenesis is
also impaired. These effects are most pronounced when administered
several days before or after wounding. 25,000 units of vitamin A daily
may reverse all these aspects, other than contraction, of wound
healing.
F. Radiation: Radiation produces abnormal fibroblasts. Collagen is also
deranged. Tissues become fibrotic. Angiogenesis is impaired. Epithelial
appendages are damaged and nuclear atypia is common in
keratinocytes. The quantity of growth factors is also limited. Lastly,
radiated tissue is predisposed to infection.

O'Leary, The Physiological Basis of Surgery, pp 103-106


Sabiston, Surgery, 14th Ed., pp 170-172

53. Increased Collagen Content in Healing Wounds


Collagen, the major component of the extracellular matrix, first
appears in healing wounds on the 3rd postinjury day when numerous
fibroblasts appear. Prior to this the wound is sealed with fibrin strands,
capillary loops, white cells, and some fibroblasts. Collagen synthesis
increases in a skin wound until it platause on the 7th day. The
synthesis/degradation ratio gradually reduces back to normal after
about 70 days post injury. The wound's strength is attributed primarily
to the crosslinking of collagen and rapidly increases during the first
two weeks. This is followed by a constant strengthening rate over 4
months and then a slowed but continued strengthening for over one
year.

Sabiston, 14th ed., page 168

54. Major Limitations to a Retrospective Clinical Study


In a retrospective clinical study, the researcher is unable to control
independent variables because they have either already occurred or
they cannot be controlled by nature. This type of research can be
referred to as investigation after the fact, research and direct control is
not possible. The researcher is unable to experimentally control or
randomly assign subjects. Retrospective research is weaker than
experimental design because of some inherent limitations.
There are three major limitations in retrospective research.
It lacks control due to:
1. Inability to randomize
www.Xerous.com
Surgery Online

31/31
2. Inability to manipulate independent variables because of its
retrospective nature.
3. The higher possibility of incorrect interpretation, thus being
misleading compared to experimental designs.

Glantz; Primer of BioStatistics 3rd edition pg. 101

55. Maturation of Incisional Scar


Scar maturation occurs during the remodeling phase of wound healing.
This is the last phase of wound healing after the inflammation stage
has been completed and beginning near the end of the proliferation
phase. Scar maturation consists of extra cellular cross-linking of
collagen fibrils (lysine and hydroxylysine crosslinks). This increases the
tensile strength of the wound. By 2 weeks, the wound has gained 7%
of its final strength, 20% by 3 weeks, and 50% by 4 weeks. The
wound continues to maturate over the following year and eventually
regains 80% of the strength of the original intact skin. Collagen
degradation is occurring during this time (due to collagenases) and is
in equilibrium with additional collagen formation.

Greenfield, Surgery, Page 60.


Sauer, Manual of Skin Diseases, Page 89

56. Molecule Suppressing Malignancy


A number of cytokines have been noted to have antitumor properties.
Interleukin-2 activates natural killer (NK) cells into lymphokine-
activated killer (LAK) cells; this therapy has an overall success rate of
20% with most success noted against renal cell carcinoma and
melanoma. The interferons produce an anti-tumor effect by increasing
expression of class I and II MHC proteins, which facilitate immune
recognition, and by activating NK cells and macrophages, and
stimulating B lymphocytes. Gamma-interferon is currently approved
for use in treatment of hairy cell leukemia. Tumor necrosis factors
(alpha and beta) have a direct lytic effect on some tumors. The exact
mechanism is not known, although the TNFs are known to stimulate
activity of many immune effector cells including NK cells and
macrophages.

O'Leary JP, The Physiologic Basis of Surgery, Williams & Wilkins, 1993,
pp 160-161
www.Xerous.com
Surgery Online

32/32

57. Pathophysiology of Overnight Fasting


After an overnight fast, liver glycogen (75 gm) is rapidly depleted, and
serum glucose falls within 15 hours, resulting in a decrease in the
secretion of insulin and increase in glucagon, cortisol, growth
hormone, and catecholamines. These changes stimulate hepatic
gluconeogenesis and glycogenolysis. Hepatic glucose production must
satisfy the energy demands of the hematopoietic system and CNS,
which are dependent on glucose oxidation during acute starvation.
The three primary gluconeogenic precursors used by the liver and
kidney for the synthesis of glucose are lactate, glycerol, and amino
acids, especially alanine and glutamine. Lactate is derived from the
Cori cycle and is released from skeletal muscle. The main
gluconeogenic precursor however is protein, with 75 g of protein
degraded daily and 8-12 g of urinary nitrogen excreted daily.
Proteolysis slows by about the 5th day of starvation and reaches a
nadir of 20 g/day. This is a result of ketoadaptation by the brain; a
switch to free fatty acids as the body's main fuel source, and a
reduction in the body's resting energy expenditure by up to 31%.
Death ensues when 30-40% of the body weight is lost.
The CNS does not start to use ketone bodies as a fuel source until late
in the course of starvation; at least 2 weeks.

Schwartz, pp 36-41 Way, pp 148-150

58. Pathophysiology of Sepsis


Most frequent causative organisms are Gram positive and Gram
negative bacteria. Antibiotics frequently control Gram positive
organisms, therefore there is a predominance of the Gram negative
septic shock processes today. Gram-positive sepsis:
S secondary to dissemination of potent exotoxin without evident
bacteremia (C.perfringens or tetani), or most often, by a fulminating
infection from staphylococcus, streptococcus, or pneumococcus
organisms. Patients with Gram positive septic shock experience
hypotension, but usually not associated with changes in cardiac and
hemodynamic parameters as seen in Gram negative shock.
Gram-negative sepsis:
Increased susceptibility in aged and those with preexisting diseases
(cardiovascular, pulmonary, hepatic and renal) Source: Most common
is genitourinary, followed by respiratory, abdomen, skin and
reproductive organs, in decreasing frequency.
Bacteriology: Most common are coliform species and anaerobic bacilli
www.Xerous.com
Surgery Online

33/33

EARLY (Normovolemic, Warm) shock:


Hypotension
High cardiac output**Normal or high CVP
Normal or increased blood volume
Low PVR
Warm, dry extremities
Hyperventilation
Respiratory alkalosis

LATE (Hypovolemic, Cold) shock:


Hypotension
Low cardiac output
Decreased blood volume
Low CVP
High PVR
Cold, cyanotic extremities
Hypoventilation
Metabolic acidosis **

The increased cardiac output is associated with a decrease in oxygen


utilization, i.e. a narrowed A-VO2 difference - Secondary to A-V
shunting or possibly a primary cellular defect in the utilization of O2.
Hyperventilation, respiratory alkalosis, and an altered sensorium may
be the first signs of Gram negative sepsis!!!

Schwartz, 6th Ed., pp 140-43


Miller, Physiologic Basis, pp 49-53 Demling, Surgical Critical Care, pp
172-182

59. Presence of C.difficile in Stool of Neonates


Clostridium difficile is part of the normal flora of 50-70% of
asymptomatic neonates and 20-50% of infants, as compared with 3%
of adults. Toxigenic strains (which produce both toxins A and B) may
be implicated in antibiotic-associated pseudomembranous colitis in the
pediatric population as well as in adults. Diarrhea due to C.difficile
infection may be a prominent symptom in neonates with
Hirschsprung's disease.

Nelson Textbook of Pediatrics, 14th ed., WB Saunders, 1992, pp 751,


954
www.Xerous.com
Surgery Online

34/34

60. The Primary Substrate(s) for Gluconeogenesis


Gluconeogenesis is defined as the formation of glucose from a large
variety of non-carbohydrate substrates. The main carbon source is
derived from amino acids. However, other sources include lactate,
pyruvate, propionate and glycerol. All amino acids except leucine and
lysine can supply carbon for gluconeogenesis. Diminished
carbohydrates in the cells and decreased blood sugar are the basic
stimuli that induce an increase in the rate of gluconeogenesis. The
main tissues that use glucose as their primary substrate are the liver
and kidney, others include the brain, red blood cells and testes.

Textbook of Biochemistry, 3rd Edition, Thomas Devlin editor, pg 324.


Biochemistry, A Case Oriented Approach, 5th Edition, pg 309,
Montgomery, Conway, Spector

61. Production of TNF in Gram-Negative Bacteremia


Gram-negative infections have a much worse prognosis than gram-
positive infections because of endotoxins released by gram-negative
bacteria. The endotoxin, which represents the lipopolysaccharide outer
membrane of these bacteria usually contains the lipid A moiety, which
is an important component for virulence. TNF is produced primarily by
T lymphocytes and macrophages in response to a variety of stimuli,
including IL-1, interferon, lipopolysaccharide, viruses, and BCG. In
animals receiving intravenous bacterial endotoxin and in patients in
septic shock, large quantities of TNF are produced. In gram-negative
infections, a stimulated macrophage produces TNF. TNF serves the
important autocrine function of self-stimulation of the monocytes and
macrophages to their full states of activation. This self-stimulated
state fully activates the production of a vast array of cytokine and
inflammatory protein signals.
The paracrine function of TNF is to stimulate neutrophils to full
activation.

O'Leary, The Physiologic Basis of Surgery, Williams & Wilkins, 1993

62. Protein Requirements in TPN


Basic Nutritional Needs
Determination of the patient's protein requirement is the first critical
step in formulating a nutritional support program. The requirements
for a normal, active man are 0.9-1.5 g protein/kg body weight daily
www.Xerous.com
Surgery Online

35/35
(6.25 g protein - 1 g nitrogen). Requirements change with the clinical
state, decreasing to 1 g/kg/d early in refeeding after starvation and
increasing to 2-3 g/kg/d in burned or severely septic patients. Total
intake may have to be limited to 40-50 g/d in hepatic failure.
Nitrogen/non-protein calories = 1:100-1:200 to obtain optimal
nitrogen balance.

Lawrence, Essentials of General Surgery, 1988, pp 72

63. Purpose of Crossmatch in Renal Transplant


Crossmatch refers to any test for the detection of sensitization against
donor histocompatibility antigens HLA. Presensitization to HLA
antibodies does not occur naturally but is the response to a previous
exposure to foreign HLA from blood transfusions, pregnancy, or prior
transplantation Many potential transplant candidates have preformed
antibodies to a potential renal allograft donor. Crossmatching to
ascertain presence of these antibodies to donor leukocytes must be
carried out immediately prior to transplantation. Hyperacute or
accelerated rejection frequently occurs if transplantation is undertaken
between recipient and donor with a strongly positive crossmatch (eg: a
positive HLA I IgG crossmatch). It should be noted that ABO
compatibility must be present or rejection will occur regardless of HLA
compatibility.
The various formed antibodies can be detected by different methods:
1. Complement-dependent lymphocytoxicity crossmatch - All utilize
recipient serum, donor cells (T cells, B cells, or monocytes), and
complement. If specific donor antibodies are present, antibody binding
leads to fixation of complement and lysis of donor lymphocytes. Most
commonly used crossmatch tests utilize mixed lymphocyte populations
(most being T cells), therefore a strong crossmatch indicates
antibodies to HLA I.
2. Flow cytometery (FC) - much more specific and sensitive.

Hanto DW, Mohanakumar T. Transplantation and Immunology. In:


Greenfield et al eds, Surgery: Scientific Princ. & Pract, Philadelphia, JB
Lipp, 1993, 470-472
Simmons RL, Ildstad ST, Smith CR, et al, Transplantation. In:Schwartz,
6th ed., p. 439

64. Radiation Resistance of Large Thoracic Tumors


The amount of radiation need to control a tumor is directly related to
www.Xerous.com
Surgery Online

36/36
its size, among other factors. A large tumor requires a larger amount of
radiation and is more resistant to radiation therapy. Therefore, surgical
reduction, or tumor debulking, prior to radiation therapy is often used
with large tumors to decrease the tumor size and the amount of
radiation therapy needed to be effective.

Cox, Moss' Radiation Oncology, 1993, p.70

65. Regulation of Antidiuretic Hormone


ADH is released from the posterior pituitary in response to a decreased
blood volume as mediated by baroreceptors in the left atrium. With a
decreased blood volume and a decreased left atrial pressure, a neural
feedback system increases the release of ADH. ADH acts on the
collecting duct in the kidney by increasing permeability of the duct and
allowing reabsorption of free water. ADH is synthesized in the
hypothalamus and follows neurons into the posterior pituitary.
An increase in plasma osmolality is the primary stimulus for ADH
secretion. Other stimuli may act directly on the posterior pituitary to
release ADH, i.e., pain, emotion, arousal, angiotensin II. The release of
ADH can occur indirectly through alterations of blood volume, plasma
osmolality and blood glucose concentrations, which are affected by
exercise, catacholamines, opiates, insulin, cortisol, and histamine.

Vander, Sherman, and Luciano, Human Physiology, 4th ed., p. 440

66. Retardation of Epithelial Neoplasia


Epidermal skin CA is the most common malignancy in humans. The
majority are linked to exposure to UV radiation and chemical agents.
Carcinogenesis in the skin can be divided into two distinct stages:
initiation and promotion.
Initiation results from a limited exposure to a specific agent, is
accomplished rapidly, and is irreversible. In the absence of further
treatment, initiated cells may not develop into tumors. However, a
large dose, or repeated doses of initiation will generally lead to tumor
formation.
Promotion requires repeated exposure to promoting agents at frequent
intervals. If the interval between exposures is prolonged, or if
promotion is terminated prior to tumor development, the effects are
reversible. Promoters induce tumor development only after initiation;
promotion in the absence of, or prior to initiation, will not result in
tumor formation. While all promoting agents induce epidermal
www.Xerous.com
Surgery Online

37/37
hyperplasia, hyperplasia alone is not sufficient for promotion.
However, hyperplasia may be a sufficient stimulus to complete
promotion if the early part of the process has been instituted by a
promoting agent.

Dermatology in General Medicine, 3rd Edition, Fitzpatrick, pg 722-23

67. Treatment of bleeding in von Willebrand's disease


Von Willebrand's disease is transmitted as an autosomal dominant. It
occurs in one in 10,000 patients. There are several types of von
Willebrand's disease. Some of these individuals have minimal bleeding
potential, while others may have major bleeding abnormalities.
The diagnosis is established by a prolonged bleeding time.
The PT and PTT will be normal.
Treatment requires replacement of von Willebrand's factor, which is a
separate protein from factor VIII. Von Willebrand's factor is
synthesized in endothelial cells throughout the body, whereas factor
VIII is produced in the liver.
Von Willebrand's factor is present in cryoprecipitate. Newer factor VIII
concentrates have a full complement of von Willebrand factor. Six
units of cryoprecipitate administered six hours prior to surgery will
frequently induce marked elevation in factor VIII production as well as
von Willebrand's factor. Patients are monitored by a bleeding time.
Due to the difficulty in isolating von Willebrand's factors as the source
of bleeding, patients are frequently treated with cryoprecipitate and
platelets if bleeding persists. In patients with known von Willebrand's
Disease, 10-40 units per kilogram of cryoprecipitate may be needed
every 12 hours to correct bleeding time. The duration of treatment
should be continued as needed, but may require 7-8 days of treatment
to control bleeding. DDAVP, an analog of the antidiuretic hormone has
been used intravenously to increase the von Willebrand factor. The
side effects of this drug include thrombocytopenia, and the release of
tissue plasminogen activator.

Schwartz, 6th Edition, p. 102


Dean, RE, "Hemostasis, Bleeding Disorders & Transfusion"

68. Site of Greatest Protein Turnover


The intake of protein for a healthy young adult is approximately 80-
120g of protein, or 13-20g of nitrogen per day. Daily fecal loss of
nitrogen is 2-3g, and 13-20g of nitrogen is lost in the urine. Following
www.Xerous.com
Surgery Online

38/38
injury, daily nitrogen excretion in the urine increases to 30-50g as urea
nitrogen representing proteolysis. The hormonal mileau of increased
cortisol, glucagon, and catecholamines, low insulin levels induce
gluconeogenesis in the liver to produce needed glucose under these
stress conditions.
Radiolabeled aminoacids incorporation studies and protein analysis
confirmed that skeletal muscle is the major source of proteolysis and
nitrogen loss. Visceral tissue, such as liver and kidney, are spared in
these stress situations. Protein turnover studies suggest that the
degree of protein metabolism and synthesis depends upon the severity
of the injury. In elective operations and minor injury, there is
decreased protein synthesis, and normal rates of protein breakdown.
With severe trauma, burns and sepsis, increased protein turnover
occurs and protein catabolism is greatly increased. This negative
nitrogen balance begins shortly after injury, and reaches a peak about
the first weak and may continue for 3-7 weeks. The negative nitrogen
balance can be reduced or eliminated by high caloric nitrogen
supplementation as with enteral or parenteral nutrition.

Schwartz et al Principles of Surgery, 6th Ed. pp 36-39

69. Wound Contamination Classification


Clean: An elective wound without inflammation made under ideal
conditions, without entry into the GI, respiratory, or oropharyngeal
tracts.
Example: Inguinal hernia repair, thyroidectomy, vascular surgery,
arthroplasty
Accepted Infection Rate: 1.5%
Clean-Contaminated: Wound entering the GI or respiratory tract
without significant spillage or presence of infection. Example:
Cholecystectomy, elective colectomy, gastrectomy
Accepted Infection Rate: 8%
Contaminated: Wounds entering the GI tract with gross spillage, or
entry into urinary tract in the presence of infected urine, or any fresh
traumatic wound.
Example: Emergent colectomy, common bile duct exploration for
cholangitis, stab wound
Accepted Infection Rate: 18%
Dirty: Wound where pus or perforated viscus is encountered, old
traumatic wounds, or grossly contaminated fresh traumatic wounds
Example: Perforated diverticulitis, perforated ulcer, open tibia-fibula
www.Xerous.com
Surgery Online

39/39
fracture
Accepted Infection Rate: > 40%

Davis et al, Clinical Surgery, 1st Edition, Vol 1, p 933

70. Characteristics of Adult Intussusception


Intussusception occurring in children is usually not associated with a
mass lesion, which leads to the intussusception. In adults, however,
benign and malignant tumors are often found at the leading edge or
base. A Meckel's diverticulum, although uncommon in adults, can also
be found. Symptoms include partial or total SBO with cramping
abdominal pain and, less often in adults, bloody diarrhea.

Sabiston, 14th Ed, Textbook of Surgery, 1991, p.803

71. Diagnosis of Afferent Limb Obstruction


Afferent limb obstruction is a complication of patients who have
undergone distal gastric resection and gastrojejunal anastomosis
(Billroth II). It is caused by obstruction at the junction of the afferent
limb coming from the duodenum to the gastric remnant.
Patients will complain of severe midepigastric pain after eating, which
is relieved by the emesis of a large amount of bile. Food is not mixed
with the emesis, which distinguishes the symptoms from those
resulting from efferent loop obstruction. The obstruction is usually
caused by recurrent ulcer or due to a technical error and re-operation
is indicated.

Schwartz, 6th ed., page 1133.

72. Diagnosis of Diarrhea with Hypokalemia


Villous adenomas of the rectum occasionally present with watery
diarrhea and hypokalemia. Diarrhea is defined as an increase in
volume, frequency or fluidity of stool output. The normal colon
receives 900-1500 ml of ileal fluid daily. All but 100-200 ml is
absorbed, most of the water and electrolytes are absorbed in the
proximal colon. Water is absorbed under the influence of an osmotic
gradient, whereas sodium is absorbed under the influence of the
sodium pump. Potassium transport is passive. The distal colon and
rectum are relatively impermeable to potassium.
There are four pathophysiologic mechanisms that lead to diarrhea:
www.Xerous.com
Surgery Online

40/40
1. Osmotic retention of intraluminal water
2. Luminal secretion of solute or water
3. Exudation
4. Lack of contact between absorptive surface and chyme.
The diarrhea associated with a villous adenoma of the rectum
represents the luminal secretion of water and potassium, which cannot
be reabsorbed by the rectum.

Sabiston, 6th Ed., pp 1026-27, 1198-99.

73. Elevated Serum Gastrin Levels


Gastrin is an intestinal polypeptide, which is produced, stored, and
released from G-cells found in antral mucosa and the mucosa of the
proximal small bowel. Gastrin's most pronounced metabolic effect is
the stimulation of gastric acid release from the gastric parietal cells.
Gastrin release is controlled by antral pH, vagal stimulation with
eating, and inhibition by the polypeptide secretin. Elevated serum
gastrin levels are present in conditions of gastrin hypersecretion
(Zollinger-Ellison syndrome, gastrinoma) and when gastrin
clearance/metabolism is diminished (chronic renal failure).

Sabiston, Textbook of Surgery, 14th Ed, pp 760-762.

74. Initial Therapy for Amoebic Liver Abscess


The initial treatment of amoebic liver abscess is conservative with the
goal of eradicating the parasite from the intestinal tract, liver and
abscess. This is now accomplished through the use of
METRONIDAZOLE. This agent has replaced chloroquine and emetine.
Hepatic and intestinal infection have been cured with 400 mg of
metronidazole TID for 4 days. This is combined with percutaneous
aspiration of the cyst if necessary. The patient generally is not
considered for surgical treatment until the intestinal phase has been
controlled. Amoebicidal therapy should precede aspiration attempts by
several days. It should be noted that in most cases, aspiration is not
required, and the abscess is effectively resolved with amoebicidal
therapy.

Schwartz SI. Liver. In: Schwartz, Shires, Spencer eds. Principles of


Surgery, 6th ed. NY, McGraw-Hill, 1994:1330.
www.Xerous.com
Surgery Online

41/41

75. Treatment of Acute Volvulus of the Sigmoid Colon


Acute volvulus of the sigmoid colon is the most common type of colonic
volvulus in the U.S., and occurs most commonly in elderly,
institutionalized patients. There is also a high incidence of colonic
volvulus in Africa. Predisposing factors are thought to be high fiber
diets with chronic constipation. The patient usually presents with
symptoms of a bowel obstruction, with generalized abdominal pain
and tenderness.
Plain films of the abdomen are diagnostic 40% of the time. An
additional 20% can be diagnosed by barium enema, with identification
of a "birds' beak" found between the rectum and the volvulized
sigmoid. The barium study is of course contraindicated if there is any
question of perforation.
Unless there are obvious signs of gangrene or perforation, the next
step in management, following stabilization, is proctoscopic or
colonoscopic decompression. Barring the presence of perforation,
gangrene, or mucosal sloughing, decompression by placement of a
long soft rectal tube is successful about 70-85% of the time. If this is
not followed up by definitive therapy, there is a recurrence rate of
33% to 90%, depending on the series.
Surgical options include sigmoid resection or sigmoidopexy. Resection
is preferred as sigmoidopexy carries a recurrence rate of 20%.
Presence of perforation or gangrene raises the operative mortality
from less than 10% when done electively after endoscopic
decompression and bowel prep, to greater than 50% when dome
emergently.

Cameron, Current Surgical Therapy - 3


Schwartz, Principles of Surgery, 5th Edition

76. Treatment of Barrett's Esophagus/High Grade Dysplasia


Barrett's esophagus is a columnar epithelium-lined esophagus
acquired as a complication of persistent reflux esophagitis. It is almost
always associated with a severe mechanical deficiency of the LES. A
patient with Barrett's esophagus is at risk for proximal progression of
the abnormality, formation of a stricture, hemorrhage from a Barrett's
ulcer, and development of adenocarcinoma.
The presence of Barrett esophagus is an indication for multiple
mucosal biopsies. If grade III dysplasia is documented by at least two
pathologists or if intramucosal carcinoma is found, esophageal
resection is indicated. If these histologic changes are not present, an
www.Xerous.com
Surgery Online

42/42
antireflux procedure should be performed in an effort to halt
progression of the disease.
Cameron provides the following:
Indications for Primary Medical Therapy:
- Obvious alterable predisposing factors, e.g., obesity, smoking
- Minimal derangement of the LES and esophageal body function
- Acid Hypersecretion
- No complications
- Localized upward extent of Barrett's mucosa
- Patient refuses surgery
- Patient is compliant with medical treatment and follow-up

Indications for Surgical Therapy:


- Perforated or bleeding Barrett's ulcer
- Severe complications, e.g., tight stricture, nonhealing ulcer, high-
grade dysplasia
- Continued Sx after medical Tx
- Increasing extent of Barrett's segment
- Development of complications during follow-up or patient preference

Indications for Esophageal Resection:


- High-grade dysplasia
- Carcinoma
- Undilatable stricture

Greenfield's Surgery pg. 616


Cameron 4th edition pg. 31

77. Treatment of Carcinoid Tumor of the Rectum


Rectal carcinoids are usually small and asymptomatic and may be seen
on proctosigmoidoscopy. Malignancy correlates with the size of the
lesion and is based on invasion into the muscularis propria.
Tumors <1 cm may be treated by endoscopic excision.
Tumors 1-2 cm should be treated by transanal excision with adequate
margins.
Tumors >2 cm may require anterior resection.
If there is invasion of the muscularis propria, anterior resection or
abdominal perineal resection should be performed. More than 60% of
rectal carcinoids >2 cm have liver metastases and 90% have local
lymph node metastases. There is also a 4% chance of malignancy in
lesions <2 cm.
www.Xerous.com
Surgery Online

43/43
Schwartz, Principles of Surgery, 6th Edition, pg 1298
Sabiston, Textbook of Surgery, 14th Edition, pg 871

78. Treatment of Complications Related to Laparoscopic


Cholecystectomy
Laparoscopic cholecystectomy is a safe and effective means of
removing the gallbladder with minimal risk. The quoted mortality is
<1% with a morbidity level of around 3%. Complications of
laparoscopic cholecystectomy are similar to the open procedure. These
include bleeding, infection, risks associated with a general anesthetic,
injury to the biliary tree and surrounding organs, and injury to vascular
structures.
However, there are inherent risks unique to laparoscopic techniques.
These include extraperitoneal insufflation, gas embolism and trocar
induced injuries. Knowledge of these complications, a high degree of
suspicion, and careful, meticulous technique can lower the incidence of
these occurrences.
Complications can be classified into two major categories:
Intraoperative Complications: Damage to the biliary tree, if recognized
intraoperatively, should be repaired primarily. This can be done
through the laparoscope or an open conversion depending on the skills
of the surgeon and the clinical indications.
Indications for conversion include (in the order of decreasing
frequency): inadequate visualization of anatomy secondary to
inflammation, bleeding, anomalies, or adhesions. Technical
complications due to loss of video may require an open approach.
Postoperative Complications: Unrecognized intraoperative
complications are amenable so long as one adheres to the general
principles of laparoscopy. These include early recognition of
complications, early repair, proper identification of biliary anatomy
prior to undertaking a reparative operation, and preoperative
catheterization of the ductal system via ERCP if indicated. The
diagnosis of injuries or complications should be made early. A high
index of suspicion is central. Unusually persistent abdominal pain, pain
out of proportion or in unlikely areas, hyperbilirubinemia and signs of
cholangitis should prompt the surgeon to search for underlying
pathology.
A HIDA scan or ultrasound of the biliary fossa is a useful adjunct in
addition to a thorough physical exam and essential blood tests.
ERCP can be useful for stenting cystic duct leaks or removing retained
stones without the need for open surgery. Furthermore, it can also aid
in diagnosis of biliary stricture or anastomotic leak.
www.Xerous.com
Surgery Online

44/44
In the case of common duct injury, a hepaticojejunostomy should be
performed.

Ballantyne, G., Laparoscopic Surgery, WB Saunders, 1994, pp 77-91,


179-181

79. Treatment of Families with Adenomatous Polyposis Coli


All patients with this disease will develop colon cancer if left
untreated.
Current recommendations include: endoscopy after 10 years of age,
with either proctoscopy or flexible sigmoidoscopy. All family members
should be screened periodically with colonoscopy from 10-40 years
old. Upper endoscopy should be completed regularly after age 30 to
screen for gastric or duodenal adenomas.
Surgical management would include: total proctocolectomy with
ileostomy, or total abdominal colectomy with ileorectal anastomosis, if
there are fewer than 50 polyps in the rectum and no rectal cancer.
Total proctocolectomy with ileal reservoir and ileoanal anastomosis is
now a reasonable consideration for these individuals. In cases where
the rectum is retained, biannual proctoscopic exams are required to
monitor these patients carefully for the development of polyps or
cancer.

Schwartz, Principles of Surgery, 6th ed., pp 1259-60

80. Treatment of Gastric Ulcer/Helicobacter Pylori Infection


Helicobacter pylori is a gram-negative micro-aerophilic curved bacillus
found in gastric biopsy specimens from patients with histologic
gastritis. The prevalence is age related with 10% incidence identified
in healthy persons younger than 30 years of age and 60% in those
over 60 yrs of age. The bacteria - H.pylori is more prevalent in patients
with duodenal or gastric ulcers, 100% and 80% respectively. H.pylori
is commonly associated with gastritis. Studies have noted increased
acid secretion and exaggerated gastrin production in patients
following a meal if infected with H. pylori. It is now generally accepted
that H. pylori causes gastritis. Data regarding H. pylori and gastric
ulcers is currently inconclusive.
Eradicating H. pylori is no easy matter!
None of the standard ulcer treatments, i.e. H2-receptor antagonist and
sucralfate, have any effect on H. pylori. Bismuth suppresses the
organism. Numerous antibiotics have been used as monotherapy
www.Xerous.com
Surgery Online

45/45
without results and resistance has developed rapidly to Tinidazole and
Metronidazole.
Currently, the recommended treatment is:
Metronidazole, 250 mg TID and Pepto-Bismol, 525 mg QID for four
weeks. Before treatment the diagnosis and sensitivity should be
confined by culture of gastric mucosal biopsy. Recent studies confirm
the close association between H.pylori, chronic gastritis and gastric
ulcer. However, clearing H.pylori infectious had no influence on the
healing or recurrence of gastric ulcers.

NEJM, Vol. 324, pp 1043-48, April 1991 JAMA, Vol. 269, pp 2934, June
1993

81. Treatment of Hepatic Metastases from Colon Carcinoma


Fifty percent of patients with gastrointestinal tumors have hepatic
metastases when autopsied. Reasonably long-term survival without
recurrence can be obtained in patients with colon cancer metastatic to
liver by resection. Primary rectal and Wilms' tumors are also good
candidates for liver resection.
Indications for hepatic resection of metastases are as follows:
1. Control of the primary tumor is accomplished or anticipated
2. There are no systemic metastases or intraabdominal carcinomatosis
3. The patient will tolerate the operative procedure
4. The extent of hepatic involvement is such that resection and total
extirpation of the metastasis is feasible.
If a hepatic metastasis is discovered during a colon resection and it is
resectable without anticipated major blood loss, it should be removed
at that time. Otherwise, resection should be delayed about two months
or more, at which time CT scan and angiography should be carried out
to assess resectability. Of the 20% of patients with colorectal cancer
having hepatic metastasis, one quarter of these are potentially
resectable, and half of these are not resection candidates because of
other metastases.
Long-term survival does not appear to be influenced by the interval
between resection of the primary lesion and resection of liver
metastases. Resection of metastatic hepatic lesion should include a
1cm margin of normal tissue, and can include resection of up to four
metastatic tumors without affecting survival.
Five year survival rates of 33% have been achieved for patients with
hepatic resections for limited hepatic metastases secondary to colon
cancer.
In non-resectable cases, palliative debulking procedures may be
www.Xerous.com
Surgery Online

46/46
indicated for control of pain associated with hepatic neoplasm.
Dearterialization and radiographically controlled embolization can be
beneficial in some cases.

Schwartz, 6th Ed., pg 1339

82. Treatment of Retained CBD Stone


Choledocholithiasis in a patient who previously underwent
cholecystectomy is best treated by endoscopic sphincterotomy
(success rate 90%, mortality is 1.0-1.5%). Extraction of stones larger
than 1.5 cm. in diameter is seldom possible and the success rate for
clearance of the duct when numerous stones are present is low when
performed by endoscope. In these situations, surgical
choledocholithotomy with choledochoduodenostomy may be the
procedure of choice. The surgical approach is also necessary in the
presence of a Billroth II reconstruction and in cases of distal common
duct stricture.
Contraindications to endoscopic removal include duodenal diverticula
in certain locations, coagulation disorders and recent pancreatitis.
Extracorporeal shock wave lithotripsy may be used when stones are
too large to extract via the endoscopic approach. If stones are noted
on T-tube cholangiogram following a common bile duct exploration, 5
general approaches are possible (with a T-tube in place and a mature
tract):
1. Small stones may be watched since the majority will remain
asymptomatic and if not, then may be surgically extracted.
2. Flushing or chemical dissolution (Capmul 8210).
3. Mechanical extraction under x-ray control (using Dormia basket, 90-
96% success).
4. Endoscopic retrieval with transduodenal papillotomy, if the T-tube
approach is unsuccessful.
5. Operative intervention.

Sabiston, Textbook of Surgery, 14th ed., pgs. 1045-1046


Schwartz, pgs. 1392-1393

83. Treatment of Villous Adenoma


Five to ten percent of colon polyps are pure villous adenomas, whereas
10-20% are tubulovillous and 65-80% are adenomatous polyps.
Villous adenomas have a 40% risk of containing a cancer, compared to
a 22% risk of cancer in tubulovillous adenomas, and 5% risk in tubular
www.Xerous.com
Surgery Online

47/47
adenomas.
The risk of cancer in a villous adenoma greater than 2 cm is 53%.
A semisessile polyps having a broad pedicle can usually be removed in
piecemeal fashion if they do not have a mottled appearance. It is
important to mark the area with India ink as a tatoo in the mucosa to
facilitate repeat colonoscopic examinations for surgical removal, if the
resected specimen shows an invasive carcinoma.
Villous adenomas of the rectum, greater than 4 cm, with areas of
induration, indicate a very high risk of cancer (90%). Because of the
high risk of sampling error, the only adequate biopsy is complete
excision, preferably transanal. The transanal excision should include
adequate peripheral margins for a complete cure, if cancer is not
present. The defect is then closed with sutures after establishing
hemostasis. There is a high risk of recurrence if inadequate excision is
performed. Occasionally, the villous adenoma will cover the entire
mucosal surface of the distal rectum. Under these circumstances, a
coloanal anastomosis after a mucosal proctectomy or full thickness
proximal proctectomy is preferred treatment. In the past, a transsacral
(Kraske) approach has been used to remove these villous adenomas as
a sphincter preserving procedure.
Villous adenomas of the small bowel are rare, and are most commonly
found in the duodenum. The presence is suspected by "soap bubble"
appearance on x-ray. They may attain a large size greater than 5 cm,
and the malignant potential is approximately 35-55%. The
pancreaticoduodenectomy procedure has been advocated as a
treatment for these lesions. Lesser procedures, including a partial
duodenectomy, can be accomplished but are frequently fraught with
duodenal or pancreatic fistulas.

Schwartz, 6th Ed., pp 1267-71, 1173

84. Site of Primary in Carcinoid Syndrome


Carcinoid tumors are most frequently identified in the appendix (46%)
followed by the ileum (28%) and rectum (17%).
A malignant potential is related to (1) location, (2) size, (3) depth of
invasion, and (4) growth pattern.
35% of the ileal carcinoids are associated with metastases, whereas
only 3% of appendiceal carcinoids metastasize. Multiple carcinoids of
the small bowel occur in 30% of the cases. Typically, ileal carcinoids
tend to be less than 1 cm. Treatment: Treatment of patients with small
bowel carcinoid tumors less than 1 cm in diameter without evidence of
regional lymph node metastases should be treated with a segmental
www.Xerous.com
Surgery Online

48/48
intestinal resection. For lesions greater than 1 cm, or patients with
multiple tumors, and the presence of regional lymph nodes, a wide
excision of the bowel and mesentery is required, and may entail a right
hemicolectomy.

Schwartz, 6th Edition, pp 1176

85. Unresectability of Carcinoma of Pancreas


Criteria of Unresectability:
1. If there are distant metastases - liver or pelvis, area of ligament of
Treitz.
2. Regional metastases - celiac axis lymph nodes and nodes posterior
to CBD
3. Local invasion - invasion of either the SMV and/or PV. Ability to
separate the anterior portion of these vessels from the posterior neck
of the pancreas is a key phase of the procedure.

Specific Intra-Operative Steps to Determine Resectability


1. Kocher maneuver - retropancreatic space is evaluated, separating
the pancreas anteriorly and vena cava/aorta posteriorly. Check
junction of the uncinate process with the SMA. Extension beyond the
pancreas to involve the retropancreatic space and SMA makes the
tumor unresectable.
2. Mobilize distal stomach and pylorus - identify structures in the
hepatogastric and hepatoduodenal ligaments: check the hepatic
artery, CBD, nodes and PV.
3. Exploration of base of the transverse mesocolon and the anterior
surface of pancreas:
- Puckering at root of mesentery indicates invasion.
- Inspect SMA, SMV, middle colic and inferior pancreaticoduodenal
vessels.
- Open lesser sac and inspect the anterior surface of the pancreas.
- Follow middle colic vessels to SMV and insert index finger from below
to elevate the neck of the pancreas.
- Insert index finger from above - ensure the space between the neck
of the pancreas and anterior surface of the portal and SMV is not
invaded.

Steps in Resection:
1. Transect common hepatic duct, retract distal biliary system and
clear the porta hepatis nodes down to celiac axis.
2. Transection of the stomach at junction of the body and antrum, add
www.Xerous.com
Surgery Online

49/49
truncal vagotomy
3. Transect neck of the pancreas just to the left of the PV.

VanHeerden, Common Problems in Cancer Surgery, pp 131-138


Cameron's Current Surgical Therapy, pp 334-337

86. Activation of Trypsinogen


Trypsinogen is the inactive precursor of trypsin. It is produced by
pancreatic acinar cells along with other inactive proteolytic enzymes,
chymotrypsinogen and procarboxypolypeptidase. These inactivated
enzymes are delivered to the duodenum in an alkaline environment.
Enterokinase, an enzyme secreted by intestinal mucosa, lyses
trypsinogen converting it into the active enzyme. Trypsin is then able
to lyse more trypsinogen into trypsin (autocatalytic activation).
Chymotrypsinogen is lysed into its active form chymotrypsin, and
procarboxypolypeptidase - activated form of trypsin also. Trypsinogen
is protected from activation prior to entering the intestinal lumen by
trypsin inhibitor. This substance is also secreted by the same
pancreatic acini cells that secrete the proteolytic enzymes.

Guyton, Medical Physiology, pg 779

87. Arterial Blood Supply of Duodenum


The main blood supply to the duodenum is from the superior and
inferior pancreaticoduodenal arteries, branches of the gastroduodenal
and superior mesenteric arteries, respectively. The proximal half of the
duodenum is supplied by the superior pancreaticoduodenal artery and
the distal half by the inferior pancreaticoduodenal artery. These
vessels anastomose to form anterior and posterior arterial arcades,
which lie in the angle between the duodenum and the pancreas.
The superior part of the duodenum may, in addition, receive blood
from:
1. The supraduodenal artery, arising from the common hepatic or
gastroduodenal,
2. The right gastric artery,
3. The right gastroepiploic artery, and
4. The gastroduodenal artery.
These vessels often anastomose with each other.

Moore, p. 220
Sabiston, p. 871
www.Xerous.com
Surgery Online

50/50

88. Characteristics of Migratory Motor Complex of the Small Bowel


Characteristic patterns of motility occur in the small intestine of
humans during fasting and after feeding. During fasting, activity
follows a cyclic pattern called the interdigestive myoelectric complex
(IDMEC) or the migrating motor complex (MMC).
Each cycle has four phases.
Phase I has little or no contractile activity or electric spike activity.
Phase II has intermittent spike activity and thus intermittent
contractile activity.
Phase III has maximum spike activity superimposed on every slow
wave; this is associated with regular, strong contractile activity.
It is usually followed by a brief period of intermittent spike activity
known as Phase IV. This serves as a transition phase between the
phase of regular contractile activity and the quiescent phase.
The duration of an entire cycle is approximately 90-120 minutes. Each
phase appears first in the distal esophagus, stomach, and duodenum
and migrates down the small intestine. The migration takes about 2
hours. Eating abolishes the interdigestive cycles and in their place
induces a pattern of intermittent contractile activity. The physiologic
significance of the MMC is not completely understood because the
complexes are present only in the fasted state and in most species
they have no apparent role in the mixing or propulsion of ingested
meals. These complexes may act as "housekeepers of the small
intestine", in the sense that they may purge the small bowel of
residual foods, secretions, and desquamated cells during the
interdigestive state. The MMC may also serve to limit the overgrowth
of bacteria in the distal small bowel. The development of Phase III of
the MMC is associated with increased secretion of pepsin and
hydrochloric acid by the stomach and of amylase and bicarbonate by
the pancreas. The association of bile secretion with the MMC has been
suggested by experiments in human subjects that have demonstrated
increasing duodenal output of bile acid and bilirubin during Phase II of
the MMC.

Miller, Physiologic Basis of Modern Surgical Care, pp 350-351

89. Characteristics of Amylase


Amylase is an enzyme responsible for the digestion of starch. The
molecule is approximately 55,000 daltons and is secreted by the
salivary glands (S-type amylase) and by the pancreas (P-type
amylase). Dietary carbohydrates consist predominately of starch,
www.Xerous.com
Surgery Online

51/51
sucrose and lactose. Digestion begins in the mouth and 30-40% is
converted to maltose, isomaltose and dextrins before S-amylase is
inactivated by gastric juice.
Pancreatic amylase completes the hydrolysis of the remaining starch in
the jejunum.
Elevated amylase levels are common with pancreatitis. Values above
100 Iu/dl are characteristic of biliary pancreatitis, lower values are
typical for acute alcoholic pancreatitis. The amylase level does not
correlate with the severity of the pancreatitis.
Normal serum amylase levels may be reported in patients with
pancreatitis if:
1. The amylase is rapidly cleared by the kidneys,
2. The pancreatic parenchyma is destroyed as in chronic pancreatitis or
3. Hyperlipidemia interferes with the amylase determination, serum
amylase levels are usually false in this setting! Elevated amylase may
also be encountered with perforated duodenum ulcer, gangrenous
cholecystitis and small bowel obstruction or infarction.

Schwartz, 6th Ed., pg 1409


O'Leary, Physiologic Basis of Surgery, p. 365

90. Characteristics of Bile Cholesterol


Bile cholesterol is predominately synthesized in the liver. The rate of
cholesterol synthesis in the liver is regulated through a negative
feedback system such that total body cholesterol synthesis is inhibited
by high cholesterol intake. Dietary cholesterol provides an insignificant
amount to the overall pool of biliary cholesterol.
Cholesterol gallstones comprise approximately 70% of all gallstones.
Maintenance of cholesterol in solution is dependent upon sufficient
amounts of bile salts and phospholipids. Alterations in this balance
result in a relative increase in the concentration of cholesterol and may
result in the precipitation of cholesterol as stones. The major vehicle
for transport and maintenance of cholesterol in solution are micelles,
which account for approximately 30% of the biliary cholesterol
transport. The remainder is carried in a vesicular form. These vesicles
have lipid bilayers similar to those found in cell membranes.
Nucleation or precipitation of cholesterol crystals, which ultimately
form macroscopic cholesterol stones is a complex phenomenon, which
may be regulated by glycoproteins found within the mucous secreted
in the gallbladder. Once cholesterol crystals have begun to form
clusters, the cholesterol stones may be an inevitable consequence.
www.Xerous.com
Surgery Online

52/52
Schwartz, 6th Edition, pp 1371, 1377

91. Characteristics of Carcinoid Tumors


Carcinoids have variable malignant potential and are composed of
multipotential cells with the ability to secrete numerous humoral
agents, the most prominent of which are serotonin and substance P.
The carcinoid syndrome, characterized by episodic attacks of
cutaneous flushing, bronchospasm, diarrhea, and vasomotor collapse,
occurs in fewer than 5% of patients with malignant carcinoids.
Carcinoids may occur in organs derived from the foregut, midgut, or
the hindgut.
In the GI tract, the appendix is the most commonly involved (46%)
followed by the ileum (28%), and the rectum (17%). The malignant
potential appears to be related to the site of origin and the size of the
primary. Only about 3% of appendiceal carcinoids metastasize, but
35% of ileal carcinoids do. The larger the size, the more the likelihood
of metastases. Carcinoids larger than 2 cm in diameter have 80-90%
incidence of malignancy.
Most common symptoms associated with ileal carcinoid are pain,
obstruction, diarrhea, and weight loss.
Primary tumors less than 1 cm without regional nodal metastases are
treated with segmental intestinal resection. For those greater than 1
cm in diameter or with regional metastases, wide excision of bowel
and mesentery is required. Malignant carcinoid tumors of the
duodenum may require radical pancreaticoduodenectomy. Carcinoid
tumors of the appendix are commonly located in the tip, thus a simple
appendectomy is curative providing the tumor is 1 cm or smaller and
without evidence of metastases. Right hemicolectomy is indicated for
tumors greater than 2 cm because of the potential for metastases.

Schwartz, 6th Edition, pp 1175-78, 1298

92. Characteristics of Cholecystokinin


Cholecystokinin (CCK) is a 33 aminoacid peptide secreted from the
duodenum and proximal small intestine.
It has several physiologic functions:
Primary Actions:
Gallbladder contraction
Relaxation of sphincter of Oddi
Stimulation of pancreatic secretin
Increased intestinal motility
www.Xerous.com
Surgery Online

53/53

Secondary Actions (Not Proven):


Inhibition of gastric emptying
Inhibition of gastric secretion
Regulation of pancreatic growth
Regulation of satiety

Stimulation of CCK secretion is produced mainly by the presence of fat


within the duodenal lumen. Only long chain fatty acids (>9 carbon
bonds) dispersed in micelles will exert this effect. Glucose, aminoacids
and other nutrients also stimulate secretion but to a lesser degree.
There are several active forms of CCK but the 33 and 8 forms are the
most active. Basal levels of CCK are low but increase 5-10 fold with
meal ingestion.
Recent studies have shown trypsin to be a major regulator of CCK
activity. Trypsin actively cleaves CCK and renders it inactive.
Experimentally, insulin potentiates CCK, whereas glucagon inhibits.
CCK binding results in activation of phospholipase C. In this second
messenger pathway, ITP-3 and DAG are formed resulting in the
intracellular increase of free calcium. Calcium then binds to calmodulin
and then activates indirectly various protein kinases responsible for
cellular function, namely, pancreatic secretion and synthesis of
enzymes.

Simmons, R. and Steed D., Basic Science Review for Surgeons, WB


Saunders, 1992, pp 244, 264-266

93. Characteristics of Heliobacter Pylori Infection


Heliobacter pylori infection of the gastroduodenal mucosa has been
implicated in the pathogenesis of gastric and duodenal ulcers. Antral
gastritis is almost always present histologically in duodenal ulcer
patients. H.pylori is almost always isolated from this antral mucosa
and the degree of inflammation strongly correlates with presence of
the organism. The antral mucosa exhibits an inflammatory response
closely resembling the changes seen in the duodenal ulcer. H.pylori is
found in 75% of patients with gastric ulceration and 50% of patients
with only dyspepsia. There is a 20% asymptomatic carrier rate and the
elderly are more likely to harbor the bacterium than are the young. It
has not been proven that H. pylori causes duodenal and gastric
ulceration. However, treatments that rid the organism from the GI
tract (colloidal bismuth-Pepto-Bismol, antibiotics-amoxicillin, biaxin,
www.Xerous.com
Surgery Online

54/54
etc.) along with H2-blockers are associated with a significantly
reduced recurrence rate than if the organism persists.

Mulholland MW. Duodenal Ulcer and Kauffman GL, Conter RL. Stress
Ulcer and Gastric Ulcer. In: Greenfield et al eds Surgery: Scientific
Principles & Practice. Philadelphia. JB Lippincott. 1993:675 and 698

94. Characteristics of Hepatocytes


The liver is comprised of three major cell types. These include Kupffer
cells or macrophages, endothelial lining cells and hepatocytes.
Hepatocytes comprise approximately 50% of the total number of cells
but take up 80% of the total volume. Hepatocytes perform most of the
metabolic and excretory functions of the liver. Arranged in one cell
layered cords or plates, their function depends on their proximity to
the hepatic blood supply.
The hepatocytes can be divided into three major zones:
Zone I cells lie in closest proximity to the periportal region and are the
first to be exposed to incoming substrates. Hence, they contain the
greatest amount of enzymes and protein substrates. They also produce
the majority of proteins and are the main cells responsible for protein
metabolism.
Zone III cells are the farthest away from the oxygen rich blood supply.
They are appropriately equipped for glycogenolysis and lipogenesis.
Ureagenesis occurs in zones II and III.
Gluconeogenesis occurs in greatest concentration in the proximal
periportal area. As blood passes from portal to central regions,
substrates and metabolites are extracted and added to the blood
accordingly. Hepatocytes regenerate extremely effectively and rapidly.
It has been estimated that the entire liver mass can be replaced every
50 days. This characteristic accounts for the rapid growth of the
hepatic remnant after major liver resections.

Simmons, R. and Steed, D., Basic Science Review for Surgeons, WB


Saunders & Co, 1992, pp 247-248

95. Characteristics of H20/Solute Absorption in the Gallbladder


The primary function of the gallbladder is to concentrate bile by
absorption of water and sodium. The gallbladder is capable of
concentrating the impermeable solutes contained in the hepatic bile by
a factor of 5-10, and reducing its volume by 80-90%. Reabsorption of
gallbladder fluid is largely determined by transport of sodium. This
www.Xerous.com
Surgery Online

55/55
cation traverses the apical (luminal) membrane via an electrochemical
gradient. An energy dependent Na-K ATPase pump maintains a low
cellular concentration of sodium by moving the sodium across the
gallbladder mucosa into the extracellular fluid, where it is in
equilibrium with the plasma. Absorption of water by the gallbladder is
a passive process and is linked to active solute transport. There is a
direct correlation between water and electrolyte absorption with time.

Schwartz, Principles of Surgery, 6th Edition, 1994, p.1369-70

96. Characteristics of Metoclopramide


I. Pharmacological activities
A. Antidopaminergic activities, like the phenothlazines, increases
gastrointestinal motility
B. Reverses narcotic-induced nausea and vomiting (i.e., postoperative
vomiting)
C. There is some evidence that it may be useful before emergency
surgery to empty the stomach
D. Tightens the lower esophageal sphincter to prevent aspiration
during emergency induction of anesthesia
E. Used in large doses in the treatment of nausea induced by
chemotherapy
F. Does NOT prevent motion-sickness
G. Useful in diabetic gastroparesis
H. Enhances response to ergotamine and analgesics in the treatment of
migraine headaches

II. Adverse reactions


A. The primary reactions are sedation and diarrhea
B. Extrapyramidal reactions, including parkinsonian symptoms and
tardive dyskinesia, have been noted when used for months or years
C. Dystonic reactions (oculogyric crises, trismus, torticollis,
opisthotonos) and akathisia are more like to occur within the first 72
hours of treatment. These are more common in children, young adults,
renal impairment, and with larger chemotherapy doses.
Extrapyramidal reactions are reversible with diphenhydramine
(Benadryl). May increase the sedative actions of CNS depressants. May
increase severity and frequency of extrapyramidal reactions produced
by other medications, particularly phenothlazines.

III. Contraindications:
1. GI obstruction, hemorrhage, or perforation
www.Xerous.com
Surgery Online

56/56
2. Convulsive disorders
3. Pheochromocytoma

IV. Pharmacokinetics:
Mostly renal excretion, some hepatic conjugation
1/2-life with normal renal function: 4-6 hours, and is not dose
dependent

Drug Evaluations, 6th ed., American Medical Association, 1986

97. Induction of Gastric Smooth Muscle Relaxation


The stomach is composed of three smooth muscular layers: an outer
longitudinal, middle circular, and inner oblique layer. The longitudinal
layers are concentrated along the lesser and greater curvatures of the
stomach. The circular muscle is present throughout the stomach and is
concentrated in a circular, muscular sphincter at the pylorus.
The vagus and sympathetic nerves are the primary effectors of smooth
muscle activity. Although the vagal nerves are considered primarily for
their motor activities, afferent neurons make up the greater part of the
cranial nerves.
Neuro control of the stomach is mediated through two gastric
myenteric plexuses (Auerbach's and Meissner's). The stomach has a
natural pacemaker located high in the greater curvature of the body of
the stomach. This initiates contractions at 3 cycles/min, as the
electrical potential passes distally, activity increases (Phase 2) and in
Phase 3 contractions are repetitive and serve as a "housekeeper
potential", which advances food toward the pylorus.
Only particles less than 1 mm in diameter are emptied into the
duodenum.
Relaxation of the smooth muscles is controlled physiologically by CCK,
mechanoreceptors for distention and glucose in the stomach and
duodenum (1st portion), which mediate relaxation via afferent fibers
to the medulla. Transient delays of gastric emptying (smooth muscle
inhibition) is seen frequently in the postoperative patient, and in those
patients with pancreatitis, peritonitis, or retroperitoneal bleeding or
injury. Gastric motility returns with resolution of the underlying
problems.
Metabolic causes for gastric atony include hypokalemia,
hypercalcemia, and hypocalcemia, hypomagnesemia, hypothyroidism,
uremia, hepatic coma, and hyperglycemia. Correction of the metabolic
abnormality usually restores normal gastric emptying.
Diabetic gastroparesis can occur in insulin-dependent diabetics. The
www.Xerous.com
Surgery Online

57/57
basic defect appears to be one of impaired neurocontrol with loss of
Phase 3 activity in the stomach. Metoclopramide is often effective in
improving gastric emptying in these patients.
Delayed gastric emptying following truncal vagotomy or gastrectomy
is a distressing problem, which is poorly understood. It may be caused
in part by the fact that the antropyloric mechanism is intact, and in
part because of concomitant sympathetectomy of the proximal
stomach.

Miller, Physiologic Basis of Modern Surgical Care, pp. 280-89

98. Complications of Ileal Resection


1. Short bowel syndrome is the fear of patients undergoing bowel
resection. Length of bowel loss leading to this syndrome varies
according to whether or not the terminal ileum and ileocecal valve is
resected.
A. Resection of 70% of the small bowel may or may not result in
malabsorption.
B. If the terminal ileum and ileocecal valve are included in the
resection, even 50% loss can result in severe malabsorption.
2. The terminal ileum is responsible for the absorption of many
substances. The most important is bile salt.
A. Loss of the active transport site for bile salts in the distal ileum
results in bile salt depletion.
B. The colon and jejunum have a limited capacity to passively absorb
bile salts.
C. The liver has a fixed capacity to compensate for losses with
increased synthesis.
D. Passage of bile salts and free fatty acids into the colon accentuate
the diarrhea problem.
E. Reduced concentrations of bile salts in bile allows cholesterol to
precipitate and the formation of gallstones.
3. Vitamin B12 is absorbed in the ileum. Chronic loss of this vitamin
can result in loss of body stores after several years.
4. Bacterial overgrowth in the small bowel may result from ileocecal
valve loss. Reflux of colonic contents into the small intestine with the
resultant bacterial overgrowth can lead to deconjugation of bile acids,
bile salts malabsorption, and bacterial metabolism of vitamin B12.
A. It is also postulated that toxic substances may be elaborated by
bacteria in the small intestine. These substances may adversely affect
intestinal motility and adaptation.
www.Xerous.com
Surgery Online

58/58
Shackelford's, Surgery of the Alimentary Tract, 3rd edition. W.B.
Saunders Co., 1991

99. Decreased Chloride Levels in Increased Pancreatic Exocrine


Secretion
The principal cations of this juice are sodium and potassium, which are
always present in concentrations similar to those found in plasma (the
sum of the two is approximately 165 mmol/L).
The concentration of the principal anions - bicarbonate and chloride
vary. When the stimulus to secrete is minimal the chloride
concentration is high (e.g. 110 mmol/L), and that of bicarbonate low -
50 mmol/L. When the secretory stimulus is maximal, the bicarbonate
concentration rises to approximately 140 mmol/L, and the chloride
concentration falls to 20 mmol/L. The variable relationship of the two
anions is a result of the passive exchange of intraductal bicarbonate
for interstitial chloride. This exchange occurs as the juice flows
through the larger pancreatic ducts on its way to the duodenum. At
slower rates, there is more opportunity for exchange to take place and
more for the bicarbonate to be lost. The alkaline pancreatic juice helps
to neutralize gastric acid in the duodenum, thus providing the optimum
pH for the activity of pancreatic digestive enzymes.

Schwartz, Principles of General Surgery, 6th Ed, p 1404

100. Exposure of the Portal Vein


The portal vein drains only the gastrointestinal tract and its unpaired
glands.
It returns the blood to the liver delivered by the celiac, superior
mesenteric, and inferior mesenteric arteries to these organs.
The portal vein is formed posterior to the neck of the pancreas by the
union of the splenic, the superior mesenteric, and the inferior
mesenteric veins. The portal vein courses posterior to the hepatic
artery and common bile duct and then ascends to the right end of the
porta hepatis where it divides into the right and left portal veins. The
right vein enters the right lobe of the liver and the left vein passes
transversely to the left end of the porta hepatis to supply the caudate,
quadrate, and left lobes.
There are no functioning valves in the portal system.
The portal vein is best exposed via an extended Kocher maneuver and
by following the superior mesenteric vein cephalad.

Moore, KL: Clinically Oriented Anatomy, p. 208, 1985.


www.Xerous.com
Surgery Online

59/59

Вам также может понравиться